Chest Wall, Pleura and Mediastinum (TCVS) Flashcards

1
Q

All of the following increase the risk or tracheal stenosis EXCEPT

A. Age over 70years

B. Radiation

C. Male gender

D. Excessive corticosteroid therapy

A

Answer: C

Intubation-related risk factors include:
1) prolonged intubation;

2) high tracheostomy through the first tracheal ring or cricothyroid membrane;
3) transverse rather than vertical incision on the trachea;
4) oversized tracheostomy tube;
5) prior tracheostomy or intubation; and
6) traumatic intubation.

Stenosis is also more common in older patients, in females, after radiation, or after excessive corticosteroid therapy, and in the setting of concomitant diseases such as autoimmune disorders, severe reflux disease, or obstructive sleep apnea and the setting of severe respiratory failure.

However, even a properly placed tracheostomy can lead to tracheal stenosis because of scarring and local injury.

Mild ulceration and stenosis are frequently seen after tracheostomy removal.

Use of the smallest tracheostomy tube possible, rapid downsizing, and a vertical tracheal incision minimize the risk for post-tracheostomy stenosis.

(See Schwartz 10th ed., p. 607.)

How well did you know this?
1
Not at all
2
3
4
5
Perfectly
2
Q

Adenoid cystic carcinomas

A. Spread submucosally

B. Exhibit aggressive growth

C. Are not radiosensitive

D. Have a 5-year survival rate of >50%

A

Answer: A

Squamous cell carcinomas often present with regional lymph node metastases and are frequently unresectable at presentation.

Their biologic behavior is similar to that of squamous cell carcinoma of the lung. Adenoid cystic carcinomas, a type of salivary gland tumor, are generally slow-growing, spread submucosally, and tend to infiltrate along nerve sheaths and within the tracheal wall.

Although indolent in nature, adenoid cystic carcinomas are malignant and can spread to regional lymph nodes, lung, and bone.

Squamous cell carcinoma and adenoid cystic carcinomas represent approximately 65% of all tracheal neoplasms.

The remaining 35% comprises small cell carcinomas, mucoepidermoid carcinomas, adenocarcinomas, lymphomas, and others.

Postoperative mortality, which occurs in up to 10% of patients, is associated with the length of tracheal resection, use of laryngeal release, the type of resection, and the histologic type of the cancer.

Factors associated with improved long- term survival include complete resection and use of radiation as adjuvant therapy in the setting of incomplete resection.

Due to their radiosensitivity, radiotherapy is frequently given post-operatively after resection of both adenoid cystic carcinomas and squamous cell carcinomas.

A dose of 50 Gray or greater is usual. Nodal positivity does not seem to be associated with worse survival.

Survival at 5 and 10 years is much better for adenoid cystic (73 and 57%, respectively) than for tracheal cancers (47 and 36%, respectively; P <0.05).

For patients with unresectable tumors, radiation may be given as the primary therapy to improve local control, but is rarely curative.

For recurrent airway compromise, stenting or laser therapies should be considered part of the treatment algorithm.

(See Schwartz 10th ed., pp. 610–611.)

How well did you know this?
1
Not at all
2
3
4
5
Perfectly
3
Q

Which of the following is NOT a non-small-cell tumor of the lung?

A. Squamous cell carcinoma

B. Adenocarcinoma

C. Carcinoid tumor

D. Large-cell carcinoma

A

Answer: C

The term non–small-cell lung carcinoma (NSCLC) includes many tumor cell types, including large cell, squamous cell, and adenocarcinoma.

Historically, these subtypes were considered to be a uniform group based on limited understanding of the distinct clinical behaviors of the subtypes as well as the act that there were few treatment options available.

With increasing understanding of the molecular biology underlying these tumor subtypes, however, the approach to diagnosis and management and the terminology used in describing these tumors is evolving rapidly.

(See Schwartz 10th ed., p. 614.)

How well did you know this?
1
Not at all
2
3
4
5
Perfectly
4
Q

The most common pattern of benign calcifcation in hamartomas is

A. Solid

B. Diffuse

C. Central

D. Popcorn

A

Answer: D

Computed tomography (CT) indings characteristic of benign lesions include small size, calcification within the nodule, and stability over time.

Four patterns of benign calcification are common: diffuse, solid, central, and laminated or “popcorn.”

Granulomatous infections, such as tuberculosis, can demonstrate the first three patterns, whereas the popcorn pattern is most common in hamartomas.

In areas of endemic granulomatous disease, differentiating benign versus malignant can be challenging.

Infectious granulomas arising from a variety of organisms account for 70 to 80% of this type of benign solitary nodules; hamartomas are the next most common single cause, accounting or about 10%.

(See Schwartz 10th ed., p. 622.)

How well did you know this?
1
Not at all
2
3
4
5
Perfectly
5
Q

For an adenocarcinoma that has pleural invasion, tumor necrosis, and has lymphovascular invasion the correct subtype is

A. Minimally invasive adenocarcinoma (MIA)

B. Lepidic predominant adenocarcinoma (LPA)

C. Invasive adenocarcinoma

D. Adenocarcinoma in situ

A

Answer: B

If lymphovascular invasion, pleural invasion, tumor necro- sis, or more than 5 mm of invasion are noted in a lesion that has lepidic growth as its predominant component, minimally invasive adenocarcinoma (MIA) is excluded, and the lesion is called lepidic predominant adenocarcinoma (LPA), and the size of the invasive component is recorded for the T stage.

(See Schwartz 10th ed., p. 615.)

How well did you know this?
1
Not at all
2
3
4
5
Perfectly
6
Q

The grade of neuroendocrine carcinoma (NEC) that is associated with hemoptysis, pneumonia, and tumor cells arranged in cords and clusters is

A. Grade VI NEC

B. Grade IV NEC

C. Grade II NEC

D. Grade I NEC

A

Answer: D

Grade I neuroendocrine carcinoma (NEC) (classic or typical carcinoid) is a low-grade NEC; 80% arise in the epithelium of the central airways.

It occurs primarily in younger patients. Because of the central location, it classically presents with hemoptysis, with or without airway obstruction and pneumonia.

Histologically, tumor cells are arranged in cords and clusters with a rich vascular stroma. This vascularity can lead to life-threatening hemorrhage with even simple bronchoscopic biopsy maneuvers.

Regional lymph node metastases are seen in 15% of patients, but rarely spread systemically or cause death.

(See Schwartz 10th ed., p. 617.)

How well did you know this?
1
Not at all
2
3
4
5
Perfectly
7
Q

Which of the following is NOT a known predictive or prognostic tumor marker for adenocarcinoma?

A. EGFR

B. KRAS

C. AFP

D. EML4-ALK

A

Answer: C

Establishing a clear histologic diagnosis early in the evaluation and management of lung cancer is critical to effective treatment.

Molecular signatures are also key determinants of treatment algorithms or adenocarcinoma and will likely become important for squamous cell carcinoma as well.

Currently, differentiation between adenocarcinoma and squamous cell carcinoma in cytologic specimens or small biopsy specimens is imperative in patients with advanced stage disease, as treatment with pemetrexed or bevacizumab-based chemotherapy is associated with improved progression-free survival in patients with adenocarcinoma but not squamous cell cancer.

Furthermore, life-threatening hemorrhage has occurred in patients with squamous cell carcinoma who were treated with bevacizumab.

Finally, EGFR mutation predicts response to EGFR tumor kinase inhibitors and is now recommended as first-line therapy in advanced adenocarcinoma.

Because adequate tissue is required for histologic assessment and molecular testing, each institution should have a clear, multidisciplinary approach to patient evaluation, tissue acquisition, tissue handling/processing, and tissue analysis.

In many cases, tumor morphology differentiates adenocarcinoma rom the other histologic subtypes. If no clear morphology can be identified, then additional testing or one immunohistochemistry marker for adenocarcinoma and one for squamous cell carcinoma will usually enable differentiation.

Immunohistochemistry for neuroendocrine markers is reserved for lesions exhibiting neuroendocrine morphology.

Additional molecular testing should be performed on all adenocarcinoma specimens for known predictive and prognostic tumor markers (eg, EGFR, KRAS, and EML4-ALK fusion gene).

Ideally, use of tissue sections and cell block material is limited to the minimum necessary at each decision point.

This emphasizes the importance of a multidisciplinary approach; surgeons and radiologists must work in direct cooperation with the cytopathologist to ensure that tissue samples are adequate or morphologic diagnosis as well as providing sufficient cellular material to enable molecular testing.

(See Schwartz 10th ed., p. 627.)

How well did you know this?
1
Not at all
2
3
4
5
Perfectly
8
Q

Desmoid tumors

A. Arise from the periosteum of the rib

B. Are treated with wide local excision with a 2- to 4-cm margin

C. Require radical excision (sacrificing neurovascular structures) to obtain 4-cm margins

D. Require chemotherapy to treat or prevent metastatic disease

A

Answer: B

Because the lesions have low cellularity and poor yield with fine needle aspiration (FNA), an open incisional biopsy or lesions over 3 to 4 cm is often necessary.

Surgery consists of wide local excision with a 2- to 4-cm margin and intraoperative frozen section assessment of resection margins.

Typically, chest wall resection, including the involved rib(s) and one rib above and below the tumor with a 4- to 5-cm margin of rib, is required.

A margin of less than 1 cm results in much higher local recurrence rates.

If a major neurovascular structure would have to be sacrificed, leading to high morbidity, then a margin of less than 1 cm would have to suffice.

Survival after wide local excision with negative margins is 90% at 10 years.

(See Schwartz 10th ed., p. 666.)

How well did you know this?
1
Not at all
2
3
4
5
Perfectly
9
Q

A 57-year-old non-small-cell lung cancer patient with a potentially resectable tumor found on computed tomography (CT) scan who can walk on a flat surface indefinitely without oxygen or stopping to rest, secondary to dyspnea will most likely tolerate

A. Lobectomy

B. Pneumonectomy

C. Single-lung ventilation

D. Wedge resection

A

Answer: A

Patients with potentially resectable tumors require careful assessment of their functional status and ability to tolerate either lobectomy or pneumonectomy.

The surgeon should first estimate the likelihood of pneumonectomy, lobectomy, or possibly sleeve resection, based on the CT images.

A sequential process of evaluation then unfolds.

A patient’s history is the most important tool for gauging risk. Specific questions regarding performance status should be routinely asked. If the patient can walk on a flat surface indefinitely, without oxygen and without having to stop and rest secondary to dyspnea, he will be very likely to tolerate lobectomy.

If the patient can walk up two lights of stairs (up two standard levels), without having to stop and rest secondary to dyspnea, he will likely tolerate pneumonectomy.

Finally, nearly all patients, except those with carbon dioxide (CO2) retention on arterial blood gas analysis, will be able to tolerate periods of single-lung ventilation and wedge resection.

(See Schwartz 10th ed., pp. 635–636.)

How well did you know this?
1
Not at all
2
3
4
5
Perfectly
10
Q

An “onion-peel” appearance of a rib on CT is suggestive of

A. Chondroma

B. Ewing sarcoma

C. Plasmacytoma

D. Osteosarcoma

A

Answer: B

Primitive neuroectodermal tumors (PNEs) (neuroblastomas, ganglioneuroblastomas, and ganglioneuromas) derive from primordial neural crest cells that migrate from the mantle layer of the developing spinal cord.

Histologically, PNEs
and Ewing sarcomas are small, round cell tumors; both possess a translocation between the long arms of chromosomes 11 and 22 within their genetic makeup.

They also share a consistent pattern of proto-oncogene expression and have been found to express the product of the MIC2 gene.

Ewing sarcoma occurs in adolescents and young adults who present with progressive chest wall pain, but without the presence of a mass.

Systemic symptoms of malaise and fever are often present.

Laboratory studies reveal an elevated erythrocyte sedimentation rate and mild white blood cell elevation.

Radiographically, the characteristic onion peel appearance is produced by multiple layers of periosteum in the bone formation. Evidence of bony destruction is also common.

The diagnosis can be made by a percutaneous needle biopsy or an incisional biopsy.

(See Schwartz 10th ed., p. 669.)

How well did you know this?
1
Not at all
2
3
4
5
Perfectly
11
Q

Pancoast tumors are identified as involving all of the following except:

A. The chest wall at or below the second rib.

B. Tumors of the parietal pleura or deeper structures overlying the first rib.

C. The superior sulcus.

D. The extreme apex of the chest.

A

Answer: A

Carcinoma arising in the extreme apex of the chest with associated arm and shoulder pain, atrophy of the muscles of the hand, and Horner syndrome presents a unique challenge to the surgeon.

Any tumor of the superior sulcus, including tumors without evidence for involvement of the neurovascular bundle, is now commonly known as Pancoast tumors, after Henry Pancoast who described the syndrome in 1932.

The designation is reserved for tumors involving the parietal pleura or deeper structures overlying the first rib.

Chest wall involvement at or below the second rib is not a Pancoast tumor.

Treatment is multidisciplinary; due to the location of the tumor and involvement of the neurovascular bundle that supplies the ipsilateral extremity, preserving postoperative
function of the extremity is critical. 

(See Schwartz 10th ed., p. 642.)

How well did you know this?
1
Not at all
2
3
4
5
Perfectly
12
Q

The most likely cause of aspiration pneumonia is

A. A mixture of aerobes and anaerobes

B. Aerobes only

C. Anaerobes only

D. Gram-negative bacteria

A

Answer: C

Normal oropharyngeal secretions contain many more Streptococcus species and more anaerobes (approximately 1 × 108 organisms/mL) than aerobes (approximately 1 × 107 organisms/mL).

Pneumonia that follows from aspiration, with or without abscess development, is typically poly- microbial.

An average of two to four isolates present in large numbers have been cultured from lung abscesses sampled percutaneously.

Overall, at least 50% of these infections are caused by purely anaerobic bacteria, 25% are caused by mixed aerobes and anaerobes, and 25% or fewer are caused by aerobes only.

In nosocomial pneumonia, 60 to 70% of the organisms are gram-negative bacteria, including Klebsiella pneumoniae, Haemophilus influenzae, Proteus species, Pseudomonas aeruginosa, Escherichia coli, Enterobacter cloacae, and Eikenella corrodens.

Immunosuppressed patients may develop abscesses because of the usual pathogens as well as less virulent and opportunistic organisms such as Salmonella species, Legionella species, Pneumocystis carinii, atypical mycobacteria, and fungi.

(See Schwartz 10th ed.,p.650.)

How well did you know this?
1
Not at all
2
3
4
5
Perfectly
13
Q

Laboratory evaluation of a chest wall mass showing elevated erythrocyte sedimentation rates indicates

A. Osteosarcoma

B. Plasmacytoma

C. Ewing sarcoma

D. Multiple myeloma

A

Answer: C

Laboratory evaluations are useful in assessing chest wall masses or the following:

  1. Plasmacytoma: Serum protein electrophoresis demonstrates a single monoclonal spike, which is measuring the overproduction of one immunoglobulin rom the malignant plasma cell clone.
  2. Osteosarcoma: Alkaline phosphatase levels may be elevated.
  3. Ewing sarcoma: Erythrocyte sedimentation rates may be elevated.

(See Schwartz 10th ed., able 19-18, p. 665.)

How well did you know this?
1
Not at all
2
3
4
5
Perfectly
14
Q

The most common benign chest wall tumor is

A. Chondromas

B. Osteochondromas

C. Desmoid tumors

D. Fibrous dysplasia

A

Answer: A

Chondromas, seen primarily in children and young adults, are one of the more common benign tumors of the chest wall.

They usually occur at the costochondral junction anteriorly and may be confused with costochondritis, except that a painless mass is present.

Radiographically, lesion is lobulated and radiodense; it may have diffuse or focal calcifications; and it may displace the bony cortex without penetration.

Chondromas may grow to huge sizes if left untreated.

Treatment is surgical resection with a 2-cm margin.

Large chondromas may harbor well-differentiated chondrosarcoma and should be managed with a 4-cm margin to prevent local recurrence.

(See Schwartz 10th ed., p. 666.)

How well did you know this?
1
Not at all
2
3
4
5
Perfectly
15
Q

Which of the following is an indication for surgical drainage of a lung abscess?

A. Abscess >3 cm in diameter.

B. Hemoptysis.

C. Failure to decrease in size after 1 week of antibiotic therapy.

D. Persistent ever.

A

Answer: B

Surgical drainage of lung abscesses is uncommon since drainage usually occurs spontaneously via the tracheobronchial tree.

Indications for intervention are:

  1. Failure of medical therapy
  2. Abscess under tension
  3. Abscess increasing in size during appropriate treatment
  4. Contralateral lung contamination
  5. Abscess >4–6 cm in diameter
  6. Necrotizing infection with multiple abscesses, hemoptysis, abscess
    rupture, or pyopneumothorax
  7. Inability to exclude a cavitating carcinoma
How well did you know this?
1
Not at all
2
3
4
5
Perfectly
16
Q

What percentages of chest wall masses are malignant?

A. 10–20%

B. 20–30%

C. 50–80%

D. 40–50%

A

Answer: C

Patients with chest wall tumors, regardless of etiology, typically complain of a slowly enlarging palpable mass (50–70%), chest wall pain (25–50%), or both.

Interestingly, growing masses are often not noticed by the patient until they suffer a trauma to the area.

Pain from a chest wall mass is typically localized to the area of the tumor; it occurs more often and more intensely with malignant tumors, but it can also be present in up to one-third of patients with benign tumors.

With Ewing sarcoma, fever and malaise may also be present.

Benign chest wall tumors tend to occur in younger patients (average age 26 years), whereas malignant tumors tend to be found in older patients (average age 40 years).

Overall, between 50 and 80% of chest wall tumors are malignant.

(See Schwartz 10th ed., p. 665.)

How well did you know this?
1
Not at all
2
3
4
5
Perfectly
17
Q

The population most at risk for developing active tuberculosis is

A. Elderly

B. Minorities

C. Urban residents

D. Human immunodeficiency virus (HIV) infected

A

Answer: D

Tuberculosis is a widespread problem that affects nearly one-third of the world’s population. Between 8.3 and 9 million new cases of tuberculosis and 12 million prevalent cases (range 10–13 million) were estimated worldwide in 2011 according to the World Health Organization. Only 10,521 new cases were reported to the World Health Organization in the United States in 2011.

Human immunodeficiency virus (HIV) infection is the strongest risk factor for developing active tuberculosis. The elderly, minorities, and recent immigrants are the most common populations to have clinical manifestations of inection, yet no age group, sex, or race is exempt from infection.

In most large urban centers, reported cases of tuberculosis are more numerous among the homeless, prisoners, and drug-addicted populations.

Immunocompromised patients additionally contribute to an increased incidence of tuberculosis infection, often developing unusual systemic as well as pulmonary manifestations.

(See Schwartz 10th ed., p. 654.)

How well did you know this?
1
Not at all
2
3
4
5
Perfectly
18
Q

The fungi associated with the highest mortality rate due to invasive mycoses in the United States is

A. Aspergillus

B. Cryptococcus

C. Candidia

D. Mucor

A

Answer: A

The genus Aspergillus comprises over 150 species and is the most common cause of mortality due to invasive mycoses in the United States.

It is typically acute in onset and life-threatening and occurs in the setting of neutropenia, chronic steroid therapy, or cytotoxic chemotherapy.

It can also occur in the general intensive care unit population of critically ill patients, including patients with underlying chronic obstructive pulmonary disease (COPD), postoperative patients, patients with cirrhosis or alcoholism, and post-inluenza patients, without any of these factors present.

The species most commonly responsible for clinical disease include A. fumigatus, A. flavus, A. niger, and A. terreus.

Aspergillus is a saprophytic, filamentous fungus with septate hyphae.

Spores (2.5–3 μm in diameter) are released and easily inhaled by susceptible patients; because the spores are microns in size, they are able to reach the distal bronchi and alveoli.

(See Schwartz 10th ed., p. 655.)

How well did you know this?
1
Not at all
2
3
4
5
Perfectly
19
Q

A patient presenting with a history and findings of dyspnea, wheezing, hemoptysis, and a mediastinal mass in the visceral compartment yields a diagnosis of

A. Lymphoma

B. Thymoma with myasthenia gravis

C. Mediastinal granuloma
D. Germ cell tumor

A

LYMPHOMA

  • Hx/PE: Night sweats, weight loss, fatigue, extrathoracic adenopathy, elevated erythrocyte sedimentation rate or C-reactive protein level, leukocytosis
  • Location: Any compartment

THYMOMA WITH MYASTHENIA GRAVIS

  • Hx/PE: Fluctuating weakness, early fatigue, ptosis, diplopia
  • Location: Anterior

MEDIASTINAL GRANULOMA

  • Hx/PE: Dyspnea, wheezing, hemoptysis
  • Location: Visceral (middle)

GERM CELL TUMOR

  • Hx/PE: Male gender, young age, testicular mass, elevated levels of human chorionic gonadotropin and/or α- fetoprotein
  • Location: Anterior
How well did you know this?
1
Not at all
2
3
4
5
Perfectly
20
Q

A patient with an anterior mediastinal mass and elevated serum α- etoprotein (AFP) most likely has

A. A teratoma

B. A nonseminomatous germ-cell tumor

C. A seminomatous germ-cell tumor

D. Metastatic hepatocellular carcinoma

A

Answer: B

The use of serum markers to evaluate a mediastinal mass can be invaluable in some patients. For example, nonseminomatous and seminomatous germ-cell tumors can frequently be diagnosed and often distinguished from one another by the levels of α- etoprotein (AFP) and human chorionic gonadotropin (hCG).

In over 90% of nonseminomatous germ-cell tumors, either the AFP or the hCG level will be elevated. Results are close to 100% specificic if the level of either AFP or hCG is greater than 500 ng/mL.

Some centers institute chemotherapy based on this result alone, without biopsy confirmation of the diagnosis. In contrast, the AFP level in patients with mediastinal seminoma is always normal; only 10% will have elevated hCG, which is usually less than 100 ng/mL.

Other serum markers, such as intact parathyroid hormone level or ectopic parathyroid adenomas, may be useful for diagnosing and also for intraoperatively confirming complete resection.

After successful resection of a parathyroid adenoma, this hormone level should rapidly normalize.

(See Schwartz 10th ed., p. 672

How well did you know this?
1
Not at all
2
3
4
5
Perfectly
21
Q

The primary site for male patients with malignant pleural effusions is

A. Gastrointestinal tract

B. Lung

C. Genitourinary tract

D. Melanoma

A

Answer: B

Malignant pleural effusions may occur in association with a number of different malignancies, most commonly lung cancer, breast cancer, and lymphomas, depending on the patient’s age and gender.

(p. 132 Schwartz ABSITE)

How well did you know this?
1
Not at all
2
3
4
5
Perfectly
22
Q

Eosinophilic granulomas are associated with

A. Langerhans cell histiocytosis (LCH)

B. Parasitic infections

C. Crohn disease

D. Gardner syndrome

A

Answer: A

Eosinophilic granulomas are benign osteolytic lesions. Eosinophilic granulomas of the ribs can occur as solitary lesions or as part of a more generalized disease process of the lymphoreticular system termed Langerhans cell histiocytosis (LCH).

In LCH, the involved tissue is infiltrated with large numbers of histiocytes (similar to Langerhans cells seen in skin and other epithelia), which are often organized as granulomas.

The cause is unknown. Of all LCH bone lesions, 79% are solitary eosinophilic granulomas, 7% involve multiple eosinophilic granulomas, and 14% belong to other forms of more systemic LCH.

Isolated single eosinophilic granulomas can occur in the ribs or skull, pelvis, mandible, humerus, and other sites. They are diagnosed primarily in children between the ages of 5 and 15 years.

Because of the associated pain and tenderness, they may be confused with Ewing sarcoma or with an inflammatory process such as osteomyelitis.

Healing may occur spontaneously, but the typical treatment is limited surgical resection with a 2-cm margin.

(See Schwartz 10th ed., p. 666.)

How well did you know this?
1
Not at all
2
3
4
5
Perfectly
23
Q

A chylothorax is likely to be present in a patient whose pleural fluid analysis results show a triglyceride level of

A. 80mg/100mL

B. 100mg/100mL

C. 45mg/100mL

D. 130mg/100mL

A

Answer: D

Laboratory analysis of the pleural fluid shows a high lymphocyte count and high triglyceride levels.

If the triglyceride level is greater than 110 mg/100 mL, a chylothorax is almost certainly present (a 99% accuracy rate).

If the triglyceride level is less than 50 mg/mL, there is only a 5% chance of chylothorax.

(See Schwartz 10th ed., p. 686.)

How well did you know this?
1
Not at all
2
3
4
5
Perfectly
24
Q

Osteosarcoma of the rib

A. Is considered nonoperable if pulmonary metastases are present

B. Is treated with radiation therapy before resection

C. Is treated with adjuvant chemotherapy before resection

D. Requires excision with a 6-cm margin

A

Answer: C

While osteosarcomas are the most common bone malignancy, they represent only 10 to 15% of all malignant chest wall tumors.

They primarily occur in young adults as rapidly enlarging, painful masses; however, osteosarcomas can occur in older patients as well, sometimes in association with previous radiation, Paget disease, or chemotherapy.

Radiographically, the typical appearance consists of spicules of new periosteal bone formation producing a sunburst appearance.

Osteosarcomas have a propensity to spread to the lungs, and up to one-third of patients present with metastatic disease.

Osteosarcomas are potentially sensitive to chemotherapy. Currently, preoperative chemotherapy is common. After chemotherapy, complete resection is performed with wide (4-cm) margins, followed by reconstruction.

In patients presenting with lung metastases that are potentially amenable to surgical resection, induction chemotherapy may be given, followed by surgical resection of the primary tumor and of the pulmonary metastases.

Following surgical treatment of known disease, additional maintenance chemotherapy is usually recommended.

(See Schwartz 10th ed., p. 667.)

How well did you know this?
1
Not at all
2
3
4
5
Perfectly
25
Q

Excisional biopsy of a chest wall mass is allowed if

A. Needle biopsy was nondiagnostic.

B. Imaging reveals classic appearance of a chondrosarcoma.

C. It is >3cm.
D. None of the above.

A

Answer: B

  1. Needle biopsy: Pathologists experienced with sarcomas
    can accurately diagnose approximately 90% o patients using FNA cytology. A needle biopsy (FNA or core) has the advantage of avoiding wound and body cavity contamination (a potential complication with an incisional biopsy).
  2. Incisional biopsy: If a needle biopsy is nondiagnostic, an incisional biopsy may be performed, with caveats. First, the skin incision must be placed directly over the mass and oriented to allow subsequent scar excision; skin laps and drains should be avoided. However, if the surgeon believes a hematoma is likely to develop, a drain is useful for limiting soft tissue contamination by tumor cells. At the time of definitive surgical resection, the en bloc resection includes the biopsy scar and the drain tract along with the tumor.
  3. Excisional biopsy: Any lesion less than 2.0 cm can be excised as long as the resulting wound is small enough to close primarily. Otherwise, excisional biopsy is performed only when the initial diagnosis (based on radiographic evaluation) indicates that the lesion is benign or when the lesion has the classic appearance of a chondrosarcoma (in which case, definitive surgical resection can be under- taken).

(See Schwartz 10th ed., p. 666.)

How well did you know this?
1
Not at all
2
3
4
5
Perfectly
26
Q

A 77-year-old male who is a former smoker (quit 10
years ago; smoked 1 pack per day for 50 years) with
COPD, type 2 diabetes mellitus, and CAD status post
CABG x 3 10 years ago, presents with a 4 month history of a non-productive cough and 10 pound weight loss in the past 3 months. A CXR and CT chest demonstrate a 2 x 3.1 cm mass in his right upper lung that was not present on a CXR taken 5 years earlier.

What is the best appropriate next step in the management of this patient?

A. Positron emission tomography (PET) scan

B. Cervical mediastinoscopy

C. Right upper lobectomy

D. Right upper lobectomy with mediastinal lymph
node sampling

E. Bone scan and abdominal CT scan

A

A. PET scanning uses fluorodeoxyglucose (FDG)
with radiolabeled fluorine to help detect malignant cells. Malignant cells metabolize glucose at a higher rate than normal cells, resulting in a radiolabeled metabolite of FDG, which results in a visual marker to help identify malignant cells, nodal stage, and metastases.

Surgical resection without any attempt at preoperative staging is not appropriate.

Although cervical mediastinoscopy may be an important step in assessing the stage of thÿs patients lung cancer, PET scanning may identify suspicious nodes and
distant metastasis and helps guide the surgeon on where to obtain the appropriate tissue for diagnosis.

How well did you know this?
1
Not at all
2
3
4
5
Perfectly
27
Q

Assuming this patient was diagnosed with a primary
non-small cell lung cancer (NSCLC) in the right upper lobe of his lung, which of the following is an absolute contraindication to surgical resection?

A. A post-operative predicted FEV1 of 45%

B. A malignant pleural effusion in the right chest

C. Tumor directly invading the chest wall

D. Tumor directly invading the right main bronchus 0.5 cm away from the carina

E. Tumor directly invading the mediastinal pleura

A

B. Surgery is contraindicated for Stage IIIB and IV lung cancer.

T4 lesions are classified as stage IIIB if they are associated with N2 disease.

Tumors of any size are T4 if they invade the heart, great vessels, trachea, esophagus, vertebral body, or carina. Tumors associated with malignant pleural or pericardial effusions are considered to have MIA disease, which precludes
surgical intervention.

T3 tumors include tumors of
any size that directly invade any of the following: diaphragm, chest wall, mediastinal pleura, parietal pericardium, or a tumor invading the main bronchus less than 2 cm from the carina (without involvement of the
carina).

Depending on nodal and metastatic involvement, T3 tumors may not result in a staging level of
IIIB or IV. Thus, surgery may be appropriate in select
patients presenting with T3 tumors.

Preoperative testing of pulmonary functional
reserve is essential prior to determining whether a
patient is a candidate for surgical resection. In gen¬
eral, a predicted post-operative FEVj less than 40% is thought to be a contraindication to surgical resection.

Although a post-operative predicted FEV1 of 45% is
not an absolute contraindication to surgical resection, it certainly merits thorough preoperative patient
assessment (including evaluation of comorbidities,
and predicted lung diffusion capacity). It is also important to remember that if the lung to be resected is not actually functional (secondary to collapse or bronchial
obstruction) than the post-operative predicted FEV,
should not be expected to change significantly from the preoperative FEV1.

Conversely, if the lung to be
resected happens to be functional, but the patient has
other areas of lung that are non-functional, than the
FEV1 will actually decrease more dramatically when
this functional lung is resected.

How well did you know this?
1
Not at all
2
3
4
5
Perfectly
28
Q

When performing a right upper lobectomy, after the pleura overlying the anterior hilum are divided from anterior to posterior, what is the order o structures that would need to be divided?

A. Superior pulmonary vein/Upper lobe bronchus/
Truncus anterior

B. Truncus anterior/Superior pulmonary vein/
Upper lobe bronchus

C. Superior pulmonary vein/Truncus anterior/
Upper lobe bronchus

D. Upper lobe bronchus/Truncus anterior/Superior

pulmonary vein

E. Truncus anterior/Upper lobe bronchus/Superior
pulmonary vein

A

C. Structures that would be need to be divided for a
right upper lobectomy from anterior to posterior are
the superior pulmonary vein, truncus anterior, and
upper lobe bronchus.

This is a simplified description of the most commonly encountered anatomy for the main structures seen.

During dissection the surgeon must be mindful of aberrant anatomy (often noted on preoperative imaging) and more commonly branching vessels, which may appear in different planes than listed above.

How well did you know this?
1
Not at all
2
3
4
5
Perfectly
29
Q

After resection of his tumor by a right upper lobectomy, pathology demonstrated squamous cell
carcinoma with negative margins. Lymph nodes sampled were also negative for malignancy. Which
of the following is true?

A. The patient will not need any further surveillance.

B. The only surveillance needed will be a repeat CT
chest in 2 years.

C. The most common site of relapse for NSCLC is
bone.

D. The patient will require radiation therapy for
6 months post-operatively.

E. Most recurrences occur within the first two years
after surgery.

A

E. NSCLC is most likely to relapse within the first two
years after resection (over 60%).

Thus, oncologic surveillance should be done every 3 to 4 months during this time.

Though there is no single recommendation for surveillance beyond 2 years, coordinated oncologic
surveillance should be performed long-term.

Though relapse of NSCLC is common in the bone, the most
common site of relapse for all stages of NSCLC is the
brain. Other sites of relapse include the lung (ipsilateral or contralateral), liver, and adrenal glands. There is
no indication for radiation therapy post-operatively.

How well did you know this?
1
Not at all
2
3
4
5
Perfectly
30
Q

If another patient presented with significant hyponatremia and seizures and was incidentally found to have similar sized lung mass in the right upper lobe, the patient would most likely benefit
from which of the following treatment modalities?

A. Surgical resection of the tumor alone

B. Single agent chemotherapy

C. Radiation therapy alone

D. Combination chemotherapy alone

E. Combination chemotherapy and radiation therapy

A

E. Paraneoplastic syndromes develop in as many as 40% of patients with newly diagnosed lung cancers.

Significant hyponatremia in the setting of a newly diagnosed lung mass is highly suspicious of syndrome of inappropriate antidiuretic hormone (SIADH). It is important to recognize that SIADH is most commonly associated with small cell lung cancer (SCLC).

In addition to hyponatremia, the diagnosis of SIADH is confirmed by demonstrating a
serum osmolarity <275mOsm/kg, urine osmolarity > serum osmolarity, and urine sodium > 25 meq/L.

Associated symptoms may include lethargy, nausea,
vomiting, altered mental status, and seizures.

SCLC account for D20% of all lung cancers. These tumors are known for rapid growth, and thus often present with necrosis. They commonly invade vascular and lymphatic tissue, and metastasize early.

Up to 70% of patients present with metastasis at the time of diagnosis. Given that SCLC is a systemic disease, chemotherapy is the primary medical treatment.

Combination chemotherapy with the addition of radiotherapy for locorégional control has been shown to improve survival. Additional prophylactic cranial irradiation has been shown to decrease the incidence brain metastasis, but has not been shown
to improve overall survival.

Other paraneoplastic syndromes associated with
hormone alterations should also be recognized.

Often Patients with SCLC may exhibit elevations in adrenocorticotropic hormone (ACTH). This may manifest with hypokalemia, hyperglycemia, and metabolic alkalosis.

Diagnosis can be confirmed by
measuring elevated ACTH in the blood or elevated
17-hydroxycorticosteroid levels in the urine.

How well did you know this?
1
Not at all
2
3
4
5
Perfectly
31
Q
  1. A 14-year-old boy is seen by a pediatric cardiologist because of increasing shortness of
    breath. Studies reveal increased pulmonary
    vascular resistance, left axis deviation on
    Electrocardiogram (ECG), and mitral regurgitation murmur. What is the most likely
    diagnosis?
    (A) Ostium primum defect
    (B) Tetralogy of Fallot
    (C) Right aortic arch
    (D) Ostium secundum defect
    (E) Atrioventricular canal
A
  1. (A) Ostium primum. Typically ostium primum
    in an adolescent would be diagnosed by
    increasing symptoms, increased pulmonary
    resistance, left axis on ECG, and a mitral regurgitation murmur due to a cleft mitral valve.
    Ostium secundum would cause increased pulmonary resistance later in life, not at age 14. AV
    canal is seen most commonly in Down syndrome. Right aortic arch and tetralogy of Fallot
    do not have this symptom complex.
How well did you know this?
1
Not at all
2
3
4
5
Perfectly
32
Q
  1. A cyanotic female neonate is born with transposition of the great arteries. Metabolic acidosis and hypoxemia are present and are life
    threatening. Which of the following is the best
    initial treatment?
    (A) Urgent Mustard operation
    (B) Prostaglandin E1
    (C) Atrial septotomy
    (D) Pulmonary artery banding
    (E) Prostaglandin E1 and atrial septotomy
A
  1. (E) Prostaglandin E1 and Aterial septotomy.
    Prostaglandin E1 is used to keep the ductus
    arteriousus open in transposition. Desaturated
    “systemic” blood can pass through the pulmonary circulation to be oxygenated. The aterial septotomy creates an ASD, which aids in
    saturated blood being pumped peripherally,
    decreasing the cyanosis. The mustard operation is not commonly done as the arterial
    switch operation is most common in this era,
    and in this acutely ill neonate definitive operation would not be the best initial treatment.
    Pulmonary artery banding does not apply.
How well did you know this?
1
Not at all
2
3
4
5
Perfectly
33
Q
  1. A 65-year-old man undergoes cardiac surgery
    for triple vessel coronary artery disease. What
    can he anticipate?
    (A) 95% chance his grafts will occlude after
    12 months.
    (B) 5% chance of living for 5 years.
    (C) If the internal mammary artery is used
    as a conduit, patency is increased.
    (D) Mortality if 10–20% in most centers.
    (E) Functional improvement with the
    saphenous vein graft is better than
    internal memory artery.
A
  1. (C) Internal thoracic artery. The internal thoracic artery is the conduit of choice especially
    for grafting the left anterior descending (LAD)
    artery. Arterial and venous grafts 95% of the
    time do not occlude after 12. Seventy-five percent of patients under coronary artery bypass
    graft (CABG) survive 5 years. Mortality is 2%
    or lower in most centers.
How well did you know this?
1
Not at all
2
3
4
5
Perfectly
34
Q
  1. Three months after aortic valve replacement
    with a mechanical prosthesis, a 60-year-old
    man describes malaise, and increasing shortness of breath. Examination reveals pulsus
    paradoxus. ECG shows low voltage precordially. What test is most useful for making the
    diagnosis?
    (A) Stress thallium exam
    (B) Computer Tomography (CT)
    examination of chest
    (C) Coronary angiography
    (D) Echocardiography
    (E) Serum creatinine phosphokinase (CPK)
A
  1. (D) Echocardiography. This patient has a pericardial effusion. Echocardiography is the most
    useful in making the diagnosis. CAT scan of the
    chest can be used but is not the best exam. The
    other choices do not apply.
How well did you know this?
1
Not at all
2
3
4
5
Perfectly
35
Q
5. In the patient described above urine output
decreases to 20 cc/h. Studies reveal paradoxical septal motion. What is the next course of
therapy?
(A) Expectant medical therapy
(B) Redo aortic valve surgery
(C) Left chest tube
(D) Ontra-aortic balloon
(E) Pericardial window
A
  1. (E) Pericardial window. The patient developed
    decreased cardiac output (decreasing urine
    output,)and cardiac tamponade. Emergent
    pericardial window is the treatment of choice.
    Medical therapy will result in the patient’s
    death. The other choices do not apply.
How well did you know this?
1
Not at all
2
3
4
5
Perfectly
36
Q
  1. A 58-year-old man is in cardiogenic shock in
    the emergency department after sustaining an
    acute myocardial infarction (MI). An intraaortic balloon pump (IABP) is inserted. Which
    statement is TRUE about IABP?
    (A) The balloon increases coronary
    perfusion during diastole.
    (B) The balloon increases coronary
    perfusion during systole.
    (C) The balloon increases peripheral
    resistance.
    (D) The balloon is inflated in systole and
    diastole.
    (E) The pump must be removed after
    24 hours.
A
  1. (A) IABP increases coronary perfusion during
    distole. The IABP inflates during diastole and
    propels blood into the coronary circulation.
    IABP decreases peripheral resistance and
    decreases afterload on the heart. The IABP can
    stay in the patient for longer than 24 hours.
How well did you know this?
1
Not at all
2
3
4
5
Perfectly
37
Q
  1. A 66-year-old female has had two MIs in the
    past. She is admitted to the emergency department in congestive heart failure. After admission and appropriate therapy her Holter
    monitor shows frequent PVCs and her ejection
    fraction is found to be 35%. Appropriate treatment would include which of the following?
    (A) Single chamber pacemaker
    (B) Cardioversion
    (C) Dual chamber pacemaker
    (D) Internal cardiac defibrillator (ICD)
    (E) Greenfield filter
A
  1. (D) ICD. In a patient with history of MI, congestive heart failure, and decreased ejection
    fraction coupled with frequent premature ventricular beats studies have shown that this
    subset of patient benefits from internal cardiac
    defibrillators, as the most frequent cause of
    death in these patients is sudden cardiac death
    from ventricular fibrillation. Single and dual
    chamber pacemakers are used for bradyarrythmias. The other choices do not apply.
How well did you know this?
1
Not at all
2
3
4
5
Perfectly
38
Q
  1. During a routine examination of a 30-year-old
    female actuary seeking life insurance, she is
    found to have a ventricular septal defect (VSD).
    She undergoes subsequent studies including
    ECG, chest x-ray, echocardiography, and
    Doppler ultrasound. What is the major determinant of operability in VSD?
    (A) Age of patient
    (B) Pulmonary vascular resistance
    (C) Size of the VSD
    (D) Location of the VSD
    (E) Presence of cyanosis
A
  1. (B) Increase in pulmonary vascular resistance
    causes an increased cardiac output. Small shunts
    (with a pulmonary/systemic flow ratio >1.5) do
    not require surgery but must be treated with
    prophylactic antibiotics. Larger shunts should be
    repaired, because the mortality rate exceeds 50%
    when severe pulmonary pressure (>85 mm Hg)
    occurs. Closure of the VSD in the presence of
    cyanosis with established reversal of the direction of flow (right to left) would be detrimental, carrying a very high mortality.
How well did you know this?
1
Not at all
2
3
4
5
Perfectly
39
Q
  1. At the age of 3 years, a child with a VSD
    becomes progressively short of breath and
    requires urgent surgery. What is the most
    common type of VSD (Fig. 4–1)?
    (A) Defect anterior to the crista
    supraventricular
    (B) Membranous septal defect
    (C) Posterior septal defect
    (D) Low muscular defect
    (E) Right-to-left shunt
A
  1. (B) VSD is the most common cardiac congenital abnormality and results from failure of
    fusion of the uppermost part of the interventricular septum with the aortic septum.
    Membranous septal defects account for 90% of
    VSDs. There is usually a left-to-right shunt and
    cyanosis does not occur until pulmonary
    hypertension is severe enough to reverse flow
    across the VSD. Surgery is indicated in large
    shunts only when symptoms occur and pulmonary hypertension is evident. Forty percent
    will close spontaneously in childhood.
How well did you know this?
1
Not at all
2
3
4
5
Perfectly
40
Q
  1. A 1-year-old girl is found to have a posterior
    membranous VSD. Peripheral resistance of the
    pulmonary system is 40% that of the systemic.
    How should you proceed?
    (A) Observe the child, because most VSDs
    close spontaneously.
    (B) Band the pulmonary artery and fix the
    defect at age 6.
    (C) Repair electively at age 14.
    (D) Repair electively between ages 4 and
    6 years.
    (E) Repair immediately as an emergency.
A
  1. (D) Increase in pulmonary resistance would
    require more urgent intervention. Because
    nearly half the cases of VSD in childhood will
    close spontaneously, elective surgery is deferred
    to late childhood. Banding procedures are used
    less frequently today because of the high mortality rate. If symptoms increase in severity and
    pulmonary pressure is high, more urgent intervention is indicated. If the pulmonary systolic
    pressure is over 85 mm Hg and the left-to-right
    shunt is small, surgical mortality exceeds 50%.
How well did you know this?
1
Not at all
2
3
4
5
Perfectly
41
Q
  1. At birth, the 6 weeks premature infant is
    noted to have progressive dyspnea. There is a
    continuous murmur in the pulmonic area (second left intercostal space), and cyanosis is
    absent. ECG findings are normal. An x-ray of
    the heart shows cardiomegaly, and the pulse
    is bounding. Patent ductus arteriosus (PDA)
    is diagnosed. What does treatment include?
    (A) Immediate surgical correction
    (B) Administration of indomethacin
    (C) Administration of cortisone
    (D) Renal dialysis
    (E) Endotracheal intubation in all cases
A
  1. (B) Management of compromised respiratory
    status in the premature infant with PDA
    includes fluid restriction, adequate oxygenation, attempted closure by medication with
    indomethacin, and surgical ligation (undertaken when indomethacin is contraindicated).
    Good results can be anticipated in the absence
    of other serious complications.
How well did you know this?
1
Not at all
2
3
4
5
Perfectly
42
Q
  1. During a routine preschool physical examination, the physician notes that a 3-year-old girl
    has a machinery-type murmur on auscultation
    of the chest. The pulse is bounding and palpable in the femoral and radial region of both
    sides of her body. There were no symptoms,
    and she has excellent exercise performance.
    Persistent PDA is confirmed on subsequent
    examination. The parents should be advised
    that the girl requires which of the following:
    (A) Surgical correction and closure of the
    PDA
    (B) Indomethacin
    (C) Coronary angiography
    (D) No treatment unless symptoms occur
    (E) CT scan of the heart
A
  1. (A) In full-term infants born with persistent
    PDA, the anomaly must be closed or excised
    between 6 months and 3 years of age to avoid
    cardiac complications, including endocarditis.
    In PDA, persistence of the communication
    between the pulmonary trunk and aorta
    increases pulmonary blood flow, left atrial
    flow, left ventricular flow, and ascending aorta
    flow. PDA accounts for 15% of all congenital
    cardiac abnormalities. Cyanosis does not occur
    initially, because oxygenated blood is shunted
    from the aorta to the pulmonary trunk. The
    murmur is continuous (sounds like machinery)
    and has harsh features. Its intensity is maximum over the left second intercostal space but
    radiates to the chest wall and the neck.
How well did you know this?
1
Not at all
2
3
4
5
Perfectly
43
Q
  1. At the age of 34 years, a female long-distance
    runner notes increasing dyspnea after running
    more than 10 mi. On inspection and palpation,
    a prominent right ventricular heave is noted.
    There is a loud systolic murmur in the left third
    interspace. The ECG shows right-axis deviation with right bundle branch block. An x-ray
    of the chest shows a small aortic knob. What
    sign or test will most likely reveal the cause of
    the congenital heart abnormality thought to be
    atrial septal defect?
    (A) Beading (scalloping) of the ribs on x-ray
    (B) Decreased carotid pulse
    (C) Left ventricular hypertrophy on ECG
    (D) Elevated sedimentation rate
    (E) Increased oxygen saturation gradient
    between the superior vena cava and the
    right ventricle
A
  1. (E) Cardiac catheterization is the definitive test
    for confirming the diagnosis of ASD. It quantifies the size of the shunt and confirms the
    increase in oxygen saturation between the right
    ventricle and the superior vena cava. Beading
    of the ribs is seen in coarctation, and a
    decreased carotid pulse is found in aortic stenosis. An elevated sedimentation rate occurs in
    the presence of infection such as bacterial
    endocarditis.
How well did you know this?
1
Not at all
2
3
4
5
Perfectly
44
Q
  1. The only son of a physiology instructor dies
    suddenly at the age of 12 years following worsening symptoms of tetralogy of Fallot. What
    would an autopsy reveal?
    (A) Dextroposition of the appendix
    (B) Brachiocephalic vein draining into the
    right renal vein
    (C) Inferior vena cava (IVC) draining to the
    superior mesenteric vein
    (D) Atrial Septal Defect (ASD)
    (E) Decreased vascularity of the lung field.
A
  1. (E) There is decreased vascularity of the lungs
    seen on chest x-ray. Tetralogy of Fallot includes
    VSD, right ventricular outflow obstruction,
    dextroposition of the aorta, and right ventricular hypertrophy. Tetralogy of Fallot accounts
    for over one-half the cases of congenital cyanotic heart disease.
How well did you know this?
1
Not at all
2
3
4
5
Perfectly
45
Q
  1. After suffering a streptococcal throat infection,
    a 12-year-old immigrant boy develops cardiac
    symptoms that are attributed to rheumatic
    fever. Years later, at the age of 34 he is admitted
    to the hospital with pulmonary edema. Further
    examination reveals a diastolic murmur at the
    apex and mitral stenosis is diagnosed. Before
    surgical evaluation, which of the following
    findings can be attributed to mitral stenosis?
    (A) Large left ventricle
    (B) Indentation of the middle third of the
    esophagus by an enlarged left atrium
    (C) Notching of the ribs
    (D) Bounding, full pulse
    (E) Angina pectoris
A
  1. (B) Dilation of the left atrium is the obvious complication following long-standing mitral stenosis.
    Echocardiography is the simplest and most precise method of showing enlargement of the left
    atrium. Frequently, there is a latency period of
    15–20 years before symptoms become evident.
    Important complications of mitral stenosis
    include exertional dyspnea caused by an increase
    in left atrial pressure and backup of blood with
    possible pulmonary edema, decreased cardiac
    output, atrial fibrillation, emboli (15%), and pressure in the intermediate third of the esophagus as
    seen on an esophogram after barium swallow.
    The pulse in mitral or aortic stenosis is reduced.
How well did you know this?
1
Not at all
2
3
4
5
Perfectly
46
Q
  1. A 23-year-old ballet dancer is concerned about
    the recent sudden death of a young famous
    Russian dancer on a New York stage. The
    patient seeks advice about his own risk for
    developing cardiac disease. His father died
    suddenly from ischemic heart disease at the
    age of 40. What is the most important risk
    factor that would further indicate the possibility of coronary artery heart disease?
    (A) Diabetes mellitus
    (B) Personality type
    (C) Elevated high-density lipoprotein
    (D) Elevation of total cholesterol/
    high-density lipoprotein ratio
    (E) Obesity
A
  1. (D) Elevation of total cholesterol/high-density
    lipoprotein is a useful predictor of coronary
    artery disease (CAD). Other known main risk
    factors include genetic predisposition, high
    cholesterol level, arterial hypertension, and cigarette smoking. Obesity, diabetes mellitus, and
    personality type are of probable importance as
    independent risk factors. The presence of
    elevated high-density lipoprotein is a favorable factor.
How well did you know this?
1
Not at all
2
3
4
5
Perfectly
47
Q
  1. In evaluating the risk factors involved in advising elective cholecystectomy in a 52-year-old
    man with heart disease, which of the following
    conditions should alert the surgeon to avoid
    an elective procedure?
    (A) MI 9 months earlier
    (B) Persistent nonspecific changes on ECG
    (C) Increased frequency and severity of
    attacks of angina
    (D) Elevated alkaline phosphatase levels
    (E) Hypertension controlled with diuretics
A
  1. (C) Changes in the nature of angina should alert
    the physician to the possible progression of the
    underlying cardiac status. The pain may become
    more severe and more frequent, may last longer,
    and may occur with a lesser degree of exertion.
    Nocturnal pain should likewise signal concern.
    In the face of unstable angina, 30% of patients are
    likely to develop MI within a 3-month period.
How well did you know this?
1
Not at all
2
3
4
5
Perfectly
48
Q
  1. After his first heart attack 3 years ago, a 63-
    year-old painter complained of central chest
    pains that radiated to the left arm after exercise.
    The pain was alleviated by nitroglycerin.
    Recently, he fell on a steel object and severed
    the median nerve and flexor tendons at the
    wrist. The skin was sutured but he is now
    scheduled to have a second operation that will
    require anesthesia. What is the best method to
    diagnose angina pectoris?
    (A) Cholesterol/high-density lipid ratio
    (B) Isoenzymes
    (C) Stress electrocardiography
    (D) Echocardiography
    (E) Chest x-ray
A
  1. (C) In about one-quarter of patients with angina
    pectoris, the ECG findings will be normal.
    Exercise electrocardiography will reveal STsegment depression and possibly precipitate
    symptoms if angina pectoris is present. There is
    a risk of myocardial death in patients tested, and
    patients with symptoms after minimal exertion
    and/or unstable angina are at particular risk
    with this procedure. If hypotension, ventricular
    arrhythmia, and supraventricular arrhythmia
    occur or if the ECG shows a fall in segment ST of
    over 3 mm, the test should be discontinued.
    In these cases, 201Tl scintigraphy would be
    used to detect cardiac ischemia or infarction.
    Echocardiography during supine exercise may
    be a helpful test in selected circumstances.
How well did you know this?
1
Not at all
2
3
4
5
Perfectly
49
Q
  1. Eight days after undergoing a hysterectomy, a 64-year-old woman complains of chest pain.
    After 12 hours, the internist orders tests to
    exclude MI. Which test will most likely support
    this diagnosis?
    (A) Serum glutamic oxaloacetic
    transaminase (SGOT) elevation
    (B) Increased sedimentation rate
    (C) 99mTc pyrophosphate scintigraphy
    showing a “hot spot”
    (D) Thallium 201 (201Tl) scintigraphy
    showing a (“hot spot”)
    (E) Dimethyliminodiacetic acid (HIDA) scan
A
  1. (C) 99mTc pyrophosphate scintigraphy showing a
    “hot spot.” Following injection of 99mTc pyrophosphate, scintigraphy may show a hot spot in the
    infarcted area. The hot spot is developed as the
    radiotracer forms a complex with calcium in
    necrotic tissue. The test should be requested
    within the first 18 hours following the onset of
    acute MI. It is not sensitive enough to detect
    small infarctions. Following 201Tl scintigraphy, a
    “cold spot” occurs because of hypoperfusion.
    The test is performed where exercise or dipyridamole (Persantine) injection can be given.
    SGOT levels are elevated in liver disease. The
    HIDA scan is used to exclude gallbladder disease. Cardiac enzyme levels and ECG findings
    are useful to establish a diagnosis of MI.
How well did you know this?
1
Not at all
2
3
4
5
Perfectly
50
Q
  1. After undergoing repair of a left indirect
    inguinal hernia, a 72-year-old obese man is
    admitted to the emergency department with
    severe retrosternal pain of 1-hours duration.
    The pain radiates to the medial aspect of the left
    hand. The ECG shows Q waves and an elevated ST-segment. A diagnosis of acute MI is
    established 1 hour after admission. Immediate
    management should include which of the
    following?
    (A) Thrombolytic therapy with tissue
    plasminogen activator (tPA)
    (B) Vitamin K
    (C) Ampicillin, 2 mg tid PO
    (D) Hydrochlorthiazide, 50 mg/d
    (E) Sodium, nitroprusside 0.5 mg/kg/min
A
  1. (A) Thrombolytic therapy intravenously with
    streptokinase, urokinase, or tPA is indicated
    in most patients with MI presenting early for
    treatment. This therapy, however, is effective
    only if initiated within 6 hours after the onset
    of pain in patients with acute MI. These drugs
    are fibrinogenolytic, and aspirin and heparin
    are frequently included in the anticoagulant
    protocol. Reperfusion rates of 60% can be
    anticipated; reocclusion rates of 15% usually
    occur. Vitamin K is not indicated, because it
    would increase the coagulability of blood. If
    a diuretic, such as hydrochlorothiazide, 25–50
    mg/d is indicated to treat milder hypertension, hypokalemia must be avoided.
How well did you know this?
1
Not at all
2
3
4
5
Perfectly
51
Q
  1. Following recovery from an acute MI, a 44-
    year-old embryology lecturer is discharged
    from the hospital with what instructions?
    (A) Angiogram every 3 months to evaluate
    the degree of atherosclerosis
    (B) Nitroglycerin three times a day
    (C) Digoxin
    (D) 325 mg of aspirin on alternate days
    (E) Pacemaker insertion
A
  1. (D) Studies have shown that in men over the
    age of 50, taking 1 tablet of aspirin (325 mg) on
    alternate days reduces the incidence of subsequent CAD complications. Nitroglycerin is prescribed if angina pectoris develops, and
    digoxin would be indicated if congestive heart
    faliure (CHF) is evident. Progression of atherosclerosis should be minimized by appropriate
    diet and exercise. The intake of excess of cholesterol and saturated fats in the diet causes
    changes in the vascular endothelium and
    smooth muscle proliferation, with subintimal
    fat and fibrous tissue accumulation leading to
    occlusion of the coronary arteries, their
    branches, and other arteries.
How well did you know this?
1
Not at all
2
3
4
5
Perfectly
52
Q
  1. A 63-year-old woman fell while crossing the
    street after her Thursday afternoon bridge
    game. Attempts at resuscitation for cardiac
    arrest by the emergency medical service (EMS)
    team were unsuccessful. The woman had previously been diagnosed as having aortic stenosis and left ventricular hypertrophy. In addition
    to these factors, which of the following predisposes to sudden cardiac death?
    (A) Split first heart sound
    (B) Hypokalemia
    (C) Soft murmur at left of sternum that
    varies with inspiration
    (D) Failure of the central venous pressure
    (CVP) to rise more than 1 cm H2O with 30-second pressure on the liver (hepatojugular reflux)
    (E) CVP of -1 cm H2O
A
  1. (B) Sudden cardiac death is defined as an unexpected death occurring within 1 hour after the
    beginning of symptoms in a patient who was
    previously hemodynamically stable. In asymptomatic patients presenting initially with cardiac disease, 20% will die within the first hour
    of symptoms. Electrolyte imbalance, hypoxia,
    and conduction system defect are additional
    factors that increase the risk of sudden death
    syndrome. Split first heart sound accentuated
    on inspiration occurs in normal individuals. In
    CHF, the CVP changes more than 1 cm when
    pressure is applied below the right costal
    margin to the liver (hepatojugular reflex) for a
    30-second period.
    New York Classification of Functional
    Changes in Heart Disease
    Class Limitation of Physical Activity
    I None
    II Slight
    III Marked
    IV Complete (even at rest)
How well did you know this?
1
Not at all
2
3
4
5
Perfectly
53
Q
  1. Three days after a patient underwent hip
    replacement for a fracture of the neck of the
    femur, the resident is called to examine the
    patient and notes hypotension ( 85/60 mm Hg)
    and a pulse rate of 104 beats per minute (bpm).
    Fluids are administered, but there is no improvement. The ECG shows peaked T waves
    and ST-elevation. Bedside monitoring reveals a
    cardiac index (CI) of 1.7 L/min/m2 (normal
    >2.2), stroke work index of 16 g/m2 (normal
    >30), and a pulmonary artery wedge pressure
    (PAWP) of 22 mm Hg (normal <15). Urgent
    treatment should involve which of the
    following?
    (A) Rapid hypertonic saline solution
    administration
    (B) Adrenaline
    (C) Inotropic agents and, if necessary,
    intra-aortic balloon counterpulsation
    (D) Indomethacin
    (E) Atropine
A
  1. (C) The patient described has cardiogenic
    shock due to postoperative MI. The mortality
    rate for patients who develop MI is increased to
    more than 60% if hypotensive cardiogenic
    shock also supervenes. Pathology studies of
    patients dying after such episodes reveal that
    more than 40% of the heart will have infarcted.
    Inotropic drugs such as dobutamine are used.
    If a rapid response is not obtained, intra-aortic
    balloon tamponade is provided to unload the
    left ventricle during systole and increase diastolic coronary arterial flow. Hypertonic solutions in graded amounts would be given only
    if hypovolemia is evident. Atropine and adrenaline would be contraindicated.
How well did you know this?
1
Not at all
2
3
4
5
Perfectly
54
Q
  1. A 58-year-old neurologist is admitted to the
    emergency department with persistent hypotension and shock following an acute MI. He is
    placed on an IABP. Which following statement is
    true about IABP?
    (A) The balloon is inflated during systole.
    (B) The balloon is inflated during diastole
    and systole.
    (C) The pump must be removed after
    10 minutes.
    (D) The balloon usually is inserted via the
    femoral artery.
    (E) Use of an IABP worsens diastolic
    coronary blood flow.
A
  1. (D) The balloon usually is inserted via the
    femoral artery. The balloon is inflated during
    diastole and deflated during systole. It is
    important that the balloon be adequately
    deflated during systole to avoid damage to the
    left ventricle. The pump can be used for a few
    days if required.
How well did you know this?
1
Not at all
2
3
4
5
Perfectly
55
Q
  1. While lying on the examining table before
    colonoscopy, a 68-year-old electrician notes palpitations. The colonoscopy was scheduled as a
    routine procedure following removal of a
    benign polyp 1 year earlier. He had rheumatic
    fever in infancy. His atrial rate on ECG is 450
    bpm, and his ventricular rate is 160 bpm. His
    pulse rate is 88 bpm. The left atrium is enlarged.
    Similar findings were noted 1 year ago, but he
    declined to take any medication. Treatment
    should entail which of the following?
    (A) Continue with colonoscopy
    (B) Continue with colonoscopy after
    administration of parenteral antibiotics
    (C) Immediate administration of antibiotics
    and follow-up colonoscopy at a later date
    (D) Immediate administration of
    anticoagulation and digoxin and
    follow-up colonoscopy at a later date
    (E) Immediate electrocardioversion with a
    current of 300–400 J
A
  1. (D) The major complications occurring in
    atrial fibrillation are cardiac failure, coronary
    ischemia, and emboli. Emboli may lead to
    stroke. Urgent cardioversion is required in
    patients with auricular fibrillation if heart failure, hypotension, or angina are also present.
    Immediate cardioversion is indicated in ventricular tachycardia or ventricular fibrillation. If
    treatment with lidocaine is ineffective, electrocardioversion with 100–200 J for ventricular
    tachycardia or 300–400 J for ventricular fibrillation is urgently indicated.
How well did you know this?
1
Not at all
2
3
4
5
Perfectly
56
Q
  1. During routine clinical examination of a 23-
    year-old seeking consultation to remove a mole
    on her left cheek, she develops tachycardia
    with a pulse rate of 186 bpm. Her pulse is regular and is otherwise asymptomatic. An ECG
    reveals supraventricular tachycardia. What
    should the treatment be?
    (A) Alternate pressure on the right and left
    carotid sinus
    (B) Bilateral simultaneous pressure over
    right and left carotid sinus
    (C) Deep eyeball pressure
    (D) Morphine sulfate, 4–8 mg IV, given
    cautiously
    (E) Electrical cardioversion
A
  1. (A)Alternate pressure over the carotid sinus
    for 20 seconds will end an attack of paroxysmal
    tachycardia in nearly one-half of cases. The
    procedure is contraindicated in patients who
    have had a cerebral TIA or those who have a
    carotid bruit. Bilateral simultaneous pressure
    on the carotid sinus carries an additional risk of
    stroke and must be avoided. The common
    carotid artery usually divides at the level of
    the upper border of the thyroid cartilage or
    hyoid bone (C3). The carotid sinus may be
    located either on the proximal internal carotid
    artery or distal common carotid bifurcation.
    Eyeball pressure may be effective but carries
    the risk of retina detachment. If initial measures
    are unsuccessful, the arrhythmia is treated with
    intravenous administration of verapamil or a
    similar drug. Electrocardioversion is indicated
    in severe cases, particularly if there are adverse
    symptoms caused by the tachycardia.
How well did you know this?
1
Not at all
2
3
4
5
Perfectly
57
Q
  1. After experiencing progressive chest pain for
    2 months, a surgical-supply store owner undergoes a CT scan that reveals a space-occupying
    lesion of the wall of the left atrium, which was
    confirmed to be a myxoma. There is no evidence of disease elsewhere. What would the
    next line of treatment be?
    (A) Excision of a myxoma performed with a
    bypass procedure
    (B) Excision of a myxoma performed
    without a bypass procedure
    (C) Insertion of a pacemaker
    (D) Chemotherapy
    (E) Radiotherapy
A
  1. (A) Myxomas constitute more than 50% of all
    primary cardiac tumors. They are usually polypoid and attached to the septum. Sarcomas
    constitute 20–25% of primary cardiac tumors.
    Cardiac metastases are seen in patients with
    metastatic disease.
How well did you know this?
1
Not at all
2
3
4
5
Perfectly
58
Q
  1. During examination of a 49-year-old male
    schoolteacher who presents with a swelling in
    the neck, palpation by a bounding pulse. Which
    test would be most likely to establish a possible
    cause of the underlying condition?
    (A) Funduscopic eye examination
    (B) Liver–spleen scan
    (C) Thyroid function studies
    (D) X-ray of the chest and cervical spine
    (E) Carotid sinus pressure
A
  1. (C) The pulse is bounding when the pulse pressure is magnified because of a wide difference
    between the systolic and diastolic pressure. It
    may be due to aortic incompetence, PDA, or
    noncardiac causes that result in increase in cardiac output and decreased peripheral resistance (e.g., hyperthyroidism, peripheral AV
    fistula, or anemia).
How well did you know this?
1
Not at all
2
3
4
5
Perfectly
59
Q
  1. Following a car accident, a 52-year-old lawyer
    complains of pain in the left abdomen and
    back. After arrival of the EMS team, her pulse
    rate is 84 bpm, but of small volume. She states
    that she has some cardiac condition but is
    uncertain of its nature. Which is the most likely
    cause of the small pulse volume?
    (A) Aortic stenosis
    (B) Syphilis
    (C) Hyperthyroidism
    (D) Carcinoid syndrome
    (E) Aortic incompetence
A
  1. (A) A small pulse occurs when the cardiac
    output is decreased and/or the peripheral
    resistance is increased. The pulse is reduced in
    aortic stenosis, heart failure, pulmonary hypertension, pulmonary incompetence, mitral
    stenosis, and pericardial effusion. The typical
    cardiac lesion in syphilis is aortic incompetence, which results in a forceful bounding
    pulse with a wide pulse pressure. Other noncardiac conditions that result in an increased
    pulse pressure include hyperthyroidism, carcinoid syndrome, and aortic incompetence.
60
Q
  1. Stenosis of which of the following vessels is
    associated with the highest patency rates following angioplasty or stenting?
    (A) Medial circumflex artery
    (B) Iliac artery
    (C) Superficial femoral artery
    (D) Popliteal artery
    (E) Tibial arteries
A
  1. (B) Angioplasty and stenting of the iliac vessels
    has a patency rate of 75% at 5 years; PTA and
    stenting of all other vessels has a much lower
    patency than bypass procedures. The FDA has
    only approved illiac artery stenting.
61
Q

Question 31
(A) History of angina and prior MI
(B) Left ventricular ejection fraction of
over 50%
(C) Aortic stenosis
(D) Signs of left ventricular failure
(E) Lowered jugular venous distension
(F) Minimal decrease in hematocrit
(G) Presence of groin hernia
(H) Decreased bowel motility
31. A 83-year-old retired navy general shows
improvement in claudication following aortoiliac bypass surgery. What is the factor that
would cause the greatest concern over the
possibility of developing cardiac complications? SELECT ONE.

A
  1. (D) The single most serious prognostic sign for
    adverse changes after vascular surgery is the
    presence of CHF. Every effort must be made to
    correct pulmonary congestion and improve left
    ventricular function before undertaking elective procedures. MI occurring within 3 months
    before operation carries a high mortality rate that will be reduced by delaying surgery for
    3–6 months when possible.
62
Q
(A) A double aortic arch
(B) Tetralogy of Fallot
(C) PDA
(D) Coarctation of the aorta
(E) Tricuspid atresia
(F) Umbilical caput medusa
(G) Neurofibromatosis (von
Recklinghausen’s disease)
(H) Noncyanotic ASD
(I) Spider nevi
(J) Femoral AV fistula
(K) Beading (notching) of the ribs
  1. Cerebrovascular accident occurs most often in
    which? SELECT ONE.
A
  1. (B) Cerebrovascular accident is the most important cause of death during the first year of life
    in patients with tetralogy of Fallot. Over 65% of
    patients with the tetralogy have cyanosis before
    1 year of age. These patients have more severe
    polycythemia and are particularly liable to
    develop cyanotic spells of unconsciousness,
    cerebral thrombosis, hemiplegia, and death.
    Brain abscess may develop subsequent to
    infarction and bacteria’s entering the systemic
    circulation via a right-to-left shunt.
63
Q
(A) A double aortic arch
(B) Tetralogy of Fallot
(C) PDA
(D) Coarctation of the aorta
(E) Tricuspid atresia
(F) Umbilical caput medusa
(G) Neurofibromatosis (von
Recklinghausen’s disease)
(H) Noncyanotic ASD
(I) Spider nevi
(J) Femoral AV fistula
(K) Beading (notching) of the ribs
  1. Dyspnea and dysphagia occur with what?
    SELECT ONE.
A
  1. (A) A double aortic arch implies that there are
    two arches of the aorta; one passes posterior to
    the esophagus and the other anterior to the trachea. The right side is more common than the
    left side, and usually one of the arches is
    smaller than the other. Respiratory difficulty
    with a labored type of respiration (often precipitated by feeding) usually occurs within the
    first few months of life. Dysphagia occurs less
    frequently. Treatment is required only if symptoms are troublesome.
64
Q
(A) A double aortic arch
(B) Tetralogy of Fallot
(C) PDA
(D) Coarctation of the aorta
(E) Tricuspid atresia
(F) Umbilical caput medusa
(G) Neurofibromatosis (von
Recklinghausen’s disease)
(H) Noncyanotic ASD
(I) Spider nevi
(J) Femoral AV fistula
(K) Beading (notching) of the ribs
  1. Differential pressure in right arm and right leg
    indicates what? SELECT ONE.
A
  1. (D) Coarctation of the aorta is a relatively
    common anomaly and accounts for approximately 15% of all congenital anomalies. The
    most common site of coarctation is immediately distal (within 3–4 cm) to the origin of the
    left subclavian artery. Normally, pressure in
    the lower extremity is higher than that in the
    upper extremity, but in coarctation of the aorta,
    the femoral pulses are absent or markedly
    reduced. Magnetic resonance imaging (MRI)
    (cine) of chest shows coarctation (Fig. 4–2).
65
Q
(A) A double aortic arch
(B) Tetralogy of Fallot
(C) PDA
(D) Coarctation of the aorta
(E) Tricuspid atresia
(F) Umbilical caput medusa
(G) Neurofibromatosis (von
Recklinghausen’s disease)
(H) Noncyanotic ASD
(I) Spider nevi
(J) Femoral AV fistula
(K) Beading (notching) of the ribs
  1. A child was born with congenital heart disease.
    The mother had rubella during pregnancy. The
    child has what? SELECT ONE.
A
  1. (C) In the fetus, the sixth left aortic arch diverts
    blood in the pulmonary artery away from the
    undeveloped lungs. After birth, the channel
    closes and becomes the ligamentum arteriosum.
    In rubella, a PDA may be associated with
    mental retardation and cataracts. Most cases
    of PDA occur without a clear-cut cause.
66
Q
(A) A double aortic arch
(B) Tetralogy of Fallot
(C) PDA
(D) Coarctation of the aorta
(E) Tricuspid atresia
(F) Umbilical caput medusa
(G) Neurofibromatosis (von
Recklinghausen’s disease)
(H) Noncyanotic ASD
(I) Spider nevi
(J) Femoral AV fistula
(K) Beading (notching) of the ribs
  1. Notching of ribs occurs in what? SELECT
    ONE.
A
  1. (D) In the presence of coarctation of the aorta,
    left ventricular enlargement, hypertrophy, and
    failure to develop occur. As the child grows,
    collaterals develop between the subclavian
    artery and the aorta via the intercostal and
    internal thoracic vessels. In children older than
    8 years of age, the intercostal arteries cause typical notching on the inferior margin of the ribs.
67
Q
(A) A double aortic arch
(B) Tetralogy of Fallot
(C) PDA
(D) Coarctation of the aorta
(E) Tricuspid atresia
(F) Umbilical caput medusa
(G) Neurofibromatosis (von
Recklinghausen’s disease)
(H) Noncyanotic ASD
(I) Spider nevi
(J) Femoral AV fistula
(K) Beading (notching) of the ribs
  1. Hypoplasia of the right ventricle occurs in
    what? SELECT ONE
A
  1. (E) Tricuspid atresia accounts for 5% of cyanotic
    heart disease. Blood to the lungs is maintained
    by a PDA.
68
Q

A 62-year-old black physician complains of headache,
nocturia, and dysuria of 3 weeks duration. Rectal
examination reveals a palpable mass in the prostate,
and a biopsy confirms the presence of prostatic carcinoma. He is advised to undergo prostatectomy. His
blood pressure is 160/105 mm Hg.
(A) Verapamil
(B) Propanalol (inderal)
(C) Deep eyeball pressure
(D) Hydrochlorthiazide diuretic
(E) Calcium phosphate
(F) Digoxin
(G) Cardiac catheterization
(H) Repeat blood pressure assessment in the
supine position
(I) Antihistamine

  1. The next step in management is which?
    SELECT ONE
A
  1. (H) Repeat blood pressure assessment in the
    supine position. Hypertension can be defined as
    a diastolic pressure above 90 mm Hg or systolic
    pressure above 160 mm Hg. Anxiety in an office
    setting may provide a false high reading of
    blood pressure. The pressure usually decreases
    when the individual remains seated and still for
    a short while. Essential hypertension implies
    that there is no clear associated cause to explain
    the hypertension. Approximately 10–15% of
    white adults and 20–30% of black adults in the
    United States suffer from hypertension.
69
Q

A 62-year-old black physician complains of headache,
nocturia, and dysuria of 3 weeks duration. Rectal
examination reveals a palpable mass in the prostate,
and a biopsy confirms the presence of prostatic carcinoma. He is advised to undergo prostatectomy. His
blood pressure is 160/105 mm Hg.
(A) Verapamil
(B) Propanalol (inderal)
(C) Deep eyeball pressure
(D) Hydrochlorthiazide diuretic
(E) Calcium phosphate
(F) Digoxin
(G) Cardiac catheterization
(H) Repeat blood pressure assessment in the
supine position
(I) Antihistamine

  1. The patient’s blood pressure remains elevated
    when assessment is repeated on several occasions. Investigations fail to reveal an underlying cause of hypertension. Before surgery, he
    should receive what? SELECT ONE.
A
  1. (D) Diuretics and angiotensinogen-converting
    enzyme (ACE) inhibitors are more likely to
    be effective in elderly black men presenting
    with hypertension. ACE inhibitors inhibit the renin–angiotensin–aldosterone system,
    sympathetic nervous system activity, and
    bradykinin degradation and cause an increase
    in prostaglandin (vasodilator) synthesis. PBlockers (e.g., propanalol) and calcium channel
    blockers (e.g., verapamil, nifedipine) are the
    first line of drugs chosen for young white men
    presenting with hypertension.
70
Q

While undergoing a physical examination for life
insurance purposes, a 46-year-old executive is noted
to have a harsh systolic murmur in the left third and
fourth parasternal area. Further evaluation, including echocardiography, reveals pulmonary stenosis.
(A) Right ventricular/pulmonary artery
gradient of 20 mm Hg
(B) Right ventricular/pulmonary artery
gradient of 65 mm Hg
(C) Left ventricular hypertrophy
(D) Right ventricular hypoplasia
(E) Absence of symptoms
(F) Hyperbaric oxygen
(G) Surgical correction
(H) Outflow tract (tunnel) to divert blood
from the aorta to the right ventricle
(I) Percutaneous balloon valvuloplasty

  1. The indication for surgery in pulmonary stenosis is what? SELECT ONE.
A
  1. (B) The presence of mild stenosis (valve gradient/right ventricular pulmonary artery <30
    mm Hg) in asymptomatic patients does not
    require surgical correction; such patients can
    anticipate a normal life expectancy. Moderate
    to severe stenosis (right ventricular/pulmonary
    artery gradient of 50–80 mm Hg) requires surgical correction.
71
Q

While undergoing a physical examination for life
insurance purposes, a 46-year-old executive is noted
to have a harsh systolic murmur in the left third and
fourth parasternal area. Further evaluation, including echocardiography, reveals pulmonary stenosis.
(A) Right ventricular/pulmonary artery
gradient of 20 mm Hg
(B) Right ventricular/pulmonary artery
gradient of 65 mm Hg
(C) Left ventricular hypertrophy
(D) Right ventricular hypoplasia
(E) Absence of symptoms
(F) Hyperbaric oxygen
(G) Surgical correction
(H) Outflow tract (tunnel) to divert blood
from the aorta to the right ventricle
(I) Percutaneous balloon valvuloplasty

  1. The appropriate treatment for significant pulmonary stenosis involves which? SELECT TWO
A
  1. (G, I) Percutaneous balloon valvuloplasty is
    now used in many centers as an initial
    approach to correct pulmonary stenosis. Right
    ventricular hypertrophy accounts for the
    parasternal heave noted on examination. Left
    ventricular hypertrophy does not occur consequent to pulmonary stenosis. Pulmonary stenosis was once considered rare but now accounts
    for 10% of cases of congenital heart disease.
72
Q

On the day of admission for elective cataract surgery, an 84-year-old retired bus driver is noted to
have a blood pressure of 255/120 mm Hg.

(A) Undergo cataract surgery after oral
diuretic therapy
(B) Undergo cataract surgery without
general anesthesia
(C) Be given a discharge order and referred
to the cardiology clinic
(D) Undergo electrocardioversion
(E) Be given sodium nitroprusside
intravenously
(F) Undergo a CT scan of the head
(G) Undergo central venous pressure
monitoring
(H) Undergo arterial blood gas (ABG)
measurement
42. Blood pressure assessment is repeated on two
occasions, and the same measurements are
obtained. What should he do? SELECT ONE.
A
  1. (E) Sodium nitroprusside, 0.5–10 mg/kg/min
    IV, is given to patients (such as the one here)
    presenting as an urgent hypertensive emergency (e.g., symptomatic hypertension with
    systolic blood pressure >200 mm Hg, or asymptomatic with systolic pressure >240 mm Hg).
    Sodium nitroprusside lowers blood pressure
    by causing arteriolar and venous dilation.
    Untreated hypertension may lead to cardiovascular, cerebrovascular, and renal disease. Other
    complications of hypertension include pulmonary edema, aortic dissection, progressive
    atherosclerosis, accelerated (malignant) hypertension, and, in pregnant patients, eclampsia.
73
Q
  1. A 61-year-old man with a long history of heavy
    smoking shows on computed axial tomography (CAT) scanning a right upper lobe tumor
    and enlarged paratracheal nodes. The tumor
    has been diagnosed as malignant by bronchoscopy. Your next move should be:
    (A) Esophagoscopy to rule out invasion of
    the esophagus.
    (B) Proceed with lobectomy and
    paratracheal node dissection.
    (C) Begin radiation of the tumor and
    paratracheal area.
    (D) Perform a mediastinoscopy for staging.
    (E) Wait 3 months and repeat CAT scan to
    evaluate further disease progression.
A
  1. (D) Next move should be sampling of mediatinal nodes to stage this carcinoma of the lung. If
    the nodes are positive, the patient is not a surgical candidate. He needs chemo-radiotherapy.
    Radiation to the mediastinal nodes should not
    begin without pathologic confirmation of nodal
    metastasis. Waiting constitutes malpractice.
74
Q
  1. A young man is shot at the level of the right
    sternoclavicular joint. His blood pressure is
    80/60 mm Hg, pulse 120 bpm, and a chest xray shows a right hydropneumothorax. The
    first step should be:
    (A) Insert a chest tube and observe for
    drainage.
    (B) Perform an immediate right thoracotomy.
    (C) Perform an angiogram to rule out great
    vessels injury.
    (D) Perform median sternotomy with
    extension along with right anterior
    boarder of the sternocleidomastoid
    muscle.
    (E) Perform a CAT scan with contrast, to
    evaluate extent of injury.
A
  1. (D) This patient has probably suffered a penetrating injury to the vessels of the thoracic outlet
    and/or superior mediastinum. Immediate operation is needed. This incision gives excellent
    exposure on the right and also gives access to
    both chest cavities.
75
Q
  1. A patient with a long history of smoking, diabetes, and hypertension develops a carcinoma
    of the right lung. Along the staging process he
    presents enlarged right mediastinal (paratracheal) nodes that, upon biopsy, are found to
    contain cancer cells. He is at stage:
    (A) IA N0 M0
    (B) IA N1 M1
    (C) IA N1 M0
    (D) IIIA
    (E) IV
A
  1. (D) Positive ipsilateral parathracheal nodes

defines stage IIIA.

76
Q
  1. During a car crash a young man suffers bilateral
    multiple fracture ribs. He is alert and presents
    shortness of breath. His blood pressure is
    100/60 mm Hg and chest is unstable. Treatment
    for this is:
    (A) Prolonged intubation and ventilatory
    support until rib fractures heal along
    with aggressive bronchial toilette.
    (B) Once the patient is stable, open rib fracture
    reduction and stabilization with plates.
    (C) Fracture stabiliztion, with towel clips on
    ribs and attached to weights (external
    fixation).
    (D) Avoid intubation, control pain, and
    perform aggressive bronchial toilette.
    (E) Temporary extracorporeal circulation to
    allow fractures to heal.
A
  1. (D) In the past, prolonged intubation (internal
    fixation) was performed with enthusiasm
    because the pulmonary failure was thought to
    be secondary to chest wall instability. Today is
    known that pulmonary failure and breathing
    problems are due to lung contusion and pain,
    respectively. Avoiding intubation, controlling
    pain, and performing aggressive bronchial toilette yield better results.
77
Q
  1. Immediately following a bout of pneumonia, a
    young woman develops a large pleural effusion. A chest tube is inserted and 600 mL of
    thin pus is obtained. A CAT scan shows incomplete drainage and multiple intrapleural loculations. Management of this empyema requires:
    (A) Insertion of multiple chest tubes under
    CAT guidance to drain either most or all
    loculations.
    (B) Treat the patient with antibiotics and
    continue single chest tube drainage.
    (C) Treat patient with antibiotics and continue
    single chest tube drainage waiting for a
    thick peel to develop and then proceed
    with open total lung decortication.
    (D) Proceed with thoracoscopy and
    intrapleural toilette. Break the
    loculations and place drains.
    (E) A thorough open total lung
    decortication immediately.
A
  1. (D) During the early period of the fibrinopurulent stage of empyema, thoracoscopy is the
    standard of care.
78
Q
  1. A 40-year-old woman treated for many years
    for gastroesophageal reflux develops dysphagia and weight loss. Previous esophagoscopy
    has revealed cellular atypia. An esophagoscopy
    is about to be performed. What is it most likely
    to reveal?
    (A) Leiomyoma arising from the long
    esophageal muscular layer
    (B) Squamous cell carcinoma arising from
    esophageal mucosal lining
    (C) Adenocarcinoma originated from
    islands of Barrett’s esophagus
    (D) Adenocarcinoma extending from the
    stomach
    (E) A large ulcer at the gastroesophageal
    junction
A
  1. (C)Adenocarcinoma, originated from islands of
    Barrett’s esophagus, is today the most common
    cancer of the esophagus in the United States.
79
Q
49. A young woman has suffered severe achalasia
of the lower most esophagus. Attempted dilations have failed. The best treatment is:
(A) Left thoracotomy and extensive
myotomy
(B) Resection of the gastoesophageal
junction and reanastomosis
Questions: 44–54 87
(C) Left thorcotomy, myotomy, and stomach
wrap (fundoplication)
(D) Laparoscopic myotomy and partial
fundoplication
(E) Transthoracic esophagogastrostomy
(side-to-side) anastomosis to avoid
disrupting the gastroesophageal
sphincter
A
  1. (D) Today, the standard of care for classic achalasia is laparoscopic myotomy and partial
    fundoplication.
80
Q
  1. Shortly after an esophagoscopy, the patient
    develops shortness of breath, chest pain, and
    fever. A contrast study shows extravasation of
    contrast into the left chest cavity. You should:
    (A) Perform a cervical esophagostomy,
    gastrostomy, insert a chest tube and
    begin high dose antibiotic therapy
    (B) Insert a nasogastric tube and begin high
    dose antibiotic therapy
    (C) Perform immediate left thoracotomy
    and repair the esophageal tear
    (D) Depoly an endoscopic intraesophageal
    stent to “plug the hole”
    (E) Stop all ingestion of food, insert a chest
    tube and begin high dose antibioic
    therapy
A
  1. (C) Perforation of the esophagus is associated
    with serious complications and death. Earliest
    repair is mandatory. Antibiotics would also be
    given. The other choices allow an on-going leak.
81
Q
  1. An 80-year-old woman walks into the emergency room complaining of vomiting and
    severe retrosternal pain. This has happened
    many times in the past. A nasogastric tube is
    inserted and there is immediate clinical
    improvement. On chest x-ray the tube is found
    looped in the chest. This patient has:
    (A) A large diveticulum of the
    mid-esophagus
    (B) The tube perforated the esophagus
    (C) Achalasia
    (D) A short esophasus
    (E) A gastric volvulus
A
  1. (E) The esophagus is not perforated because of
    the patient’s dramatic improvement. Achalasia
    is usually accompanied by chronic dysphagia.
    Short esophagus does not present with severe
    retrosternal pain. The nasogastric tube is
    looped inside the intrathoracic, volvulated
    stomach and the patient has improved because
    of decompression.
82
Q
  1. The best treatment of this 80-year-old woman
    with vomiting retrosternal pain and a looped
    nasogastric tube in her left chest is:
    (A) Remove the tube because the patient is
    now well and discharged.
    (B) Evaluate the esophageal myotomy to
    treat achalasia.
    (C) Immediate left thoracotomy to treat
    perforation.
    (D) Consider surgical reduction of volvulus
    and diaphragmatic repair.
    (E) Do not consider any surgical repair
    because the patient is too old.
A
  1. (D) Recurrent volvulus of the stomach into the
    chest is a serious condition that can lead to incarceration and gangrene. Every attempt should
    be made to repair this diaphragmatic hernia.
83
Q
  1. While landing at the end of flight a young
    woman develops shortness of breath and rightsided pressure chest pain. She is tall and thin.
    The pain, although less in intensity, occurs
    during her menstrual periods. She has not previously consulted a doctor. A chest film is likely
    to show?
    (A) Left pleural effusion
    (B) Pneumothorax
    (C) Dilated stomach
    (D) Widening of the mediastinum
    (E) Cardiomegaly
A
  1. (B) The presentation itself should alert the clinician to the possibility of a pneumothorax (Fig
    4–3). This condition is seen quite frequently with
    patients that are thin and tall. This lady presents
    with a catamenial pneumothorax syndrome.
84
Q
54. And the treatment is:
(A) Insertion of a chest tube
(B) Immediate cardiology consult
(C) Thoracentesis
(D) Insertion of a nasogastric tube
(E) A CAT scan
A
  1. (A) This is the first documented pneumothorax
    on this patient. The treatment of choice is insertion of a chest tube. If the air leak persists for
    more than 3 days or if she develops a recurrence after discharge, a thoracoscopy, resection
    of bullae and pleurodesis becomes the treatment of choice.
85
Q
(A) Transvalvular gradient of 50 mm or
more
(B) History of congestive heart failure
(C) Transient ischemic attacks (TIA)
(D) Angina
(E) Aortic insufficiency
(F) Aortic dissection
(G) Ventricular fibrillation
(H) Mitral insufficiency
(I) Acute MI
  1. A 50-year-old man has a systolic heart murmur
    best heard in the second interspace on the right
    side. He is increasingly short of breath. Which
    of the above clinical settings would determine
    the decision to operate? SELECT ONE.
A
  1. (A) The decision to operate in patients with
    aortic stenosis is based on transvalvular
    gradient. 50-mm gradient is termed critical
    aortic stenosis and the valve should be replaced
    in a symptomatic patient.
86
Q
(A) Transvalvular gradient of 50 mm or
more
(B) History of congestive heart failure
(C) Transient ischemic attacks (TIA)
(D) Angina
(E) Aortic insufficiency
(F) Aortic dissection
(G) Ventricular fibrillation
(H) Mitral insufficiency
(I) Acute MI
  1. A 68-year-old female with aortic stenosis needs
    a valve replacement. Which of the above might
    result in a poor result for this patient? SELECT
    ONE.
A
  1. (B) Congestive heart failure. In patients with
    aortic stenosis, risk factors include a history of
    agina, stroke or TIAs, and a history of congestive heart failure, which indicates a compromised left ventricle. Of the three, congestive
    heart failure is the factor which is the greatest
    risk factor for patients undergoing surgery.
87
Q
(A) Transvalvular gradient of 50 mm or
more
(B) History of congestive heart failure
(C) Transient ischemic attacks (TIA)
(D) Angina
(E) Aortic insufficiency
(F) Aortic dissection
(G) Ventricular fibrillation
(H) Mitral insufficiency
(I) Acute MI
  1. A 55-year-old man with a diastolic murmur
    heard in the second interspace on the right that
    radiates toward the apex of the heart. The cardiac
    index is normal at rest but decreases with exercise. The most likely diagnosis is? SELECT ONE.
A
  1. (E) Aortic insufficiency. This is the murmur of
    a patient with aortic insufficiency. Typically,
    these patients will be well compensated at rest
    but will have decreased cardiac output with
    exercise. These patients should be operated on.
88
Q
(A) Transvalvular gradient of 50 mm or
more
(B) History of congestive heart failure
(C) Transient ischemic attacks (TIA)
(D) Angina
(E) Aortic insufficiency
(F) Aortic dissection
(G) Ventricular fibrillation
(H) Mitral insufficiency
(I) Acute MI
  1. A 45-year-old tall, thin, male has acute onset of
    chest pain radiating into the back. In the emergency room his right radial pulse is bounding
    but his femoral pluses are absent. The most
    likely diagnosis is? SELECT ONE.
A
  1. (F) Aortic dissection. This describes a patient
    with Marfan syndrome, who are typically at
    risk for aortic dissection. With dissection you
    may preserve right radial pulse but lose
    femoral pulses.
89
Q
  1. A 56-year-old male has history of leg pain at
    rest. Patient also has history of severe coronary
    artery diseases. He cannot walk two flights of
    steps without getting short of breath. He underwent evaluation and was noted to have complete aortoiliac occlusive disease. He needs
    surgery. Which one of the following options is
    acceptable?
    (A) Aortobililiac bypass
    (B) Aortobifemoral bypass
    (C) Aortoiliac angioplasty and stent
    placement
    (D) Axillobifemoral bypass
    (E) Axilloiliac
A
  1. (D) The treatment goal in these patients is to
    reestablish blood flow to the lower extremity.
    The treatment is based on the findings at
    angiogram. All the treatment options are valid
    and are used in treatment of the aortoocclusive
    disease. Patients with short-segment (TASCA)
    stenosis in common iliac artery are treated with
    angioplasty and/or stent placement and the
    patency results are expected to be comparable to
    surgery. In patients with long-segment stenosis
    and good risk patient treatment options would
    include aortobifemoral bypass. These procedures
    are long lasting. The long-term patency rates are
    reported to be 65–90%. Axillobifemoral bypass is
    utilized in patients with high risk and poor general condition. The patency rates for this group
    vary between 50–85% in 5 years. The patient
    described would be an ideal candidate for axillobifemoral bypass.
90
Q
  1. A 65-year-old female on her routine examination was noted to have a pulsatile abdominal
    mass. She has been otherwise healthy with history of hypertension with no other history,
    except family history of father dying of ruptured abdominal aortic aneurysm. What are
    the acceptable reasons to operate on abdominal
    aortic aneurysms in 65-year-old female with
    5-cm infrarenal aneurysm?
    (A) Presence of aneurysm
    (B) Aneurysm with intramural thrombus
    (C) Asymptomatic aneurysm 5.5 cm
    (D) Associated 2-cm iliac aneurysm
    (E) Patient with splenic artery aneurysm
    1.5 cm
A
  1. (C) The current indication for repair of abdominal aortic aneurysm in female includes
    aneurysm size 5 cm in acceptable risk patient. A
    United Kingdom small aneurysm study has
    increased the size that could be observed to
    5.5 cm in male while in female it is acceptable
    to treat aneurysm at 5 cm size for acceptable
    risk. Any aneurysm with associated complication should be treated; just the presence of
    intramural thrombus does not justify repair.
    Asymptomatic 5.5-cm aneurysm should be
    treated in all patients, male or female, at acceptable cardiac risk. Patients with 2-cm aneurysm
    of iliac artery without any symptoms and complications should be observed; as the risk of surgery is higher than risk of observation till they
    reach to 4 cm. In patients, not in child-bearing
    age, 1.5-cm splenic aneurysm could be observed.
91
Q
  1. An 89-year-old male presents with asymptomatic 8-cm abdominal aneurysm. He has a
    recent history of myocardial infarction (MI) and
    is not a candidate for coronary artery bypass.
    What should the treatment options include?
    (A) Conservative treatment observation
    (B) Computerized axial tomography (CAT)
    scan to evaluate eligibility for
    endovascular repair
    (C) Open repair without any further
    workup
    (D) Axillofemoral bypass and coil
    embolization of aneurysm
    (E) b-blocker therapy
A
  1. (B) An 8-cm aneurysm carries significant
    mortality which exceeds 50% in 1 year from
    aneurysm related death if observation or medical management is chosen as treatment option.
    It would be appropriate, if the neck size is
    greater than 1.5 cm and diameter is less than
    26 mm, without any significant thrombus or
    calcification in the neck. This patient does well
    at least on mid term follow-up. They have
    lower perioperative morbidity compared to traditional open repair. Open repair with given
    cardiac history would carry high morbidity
    and morotality. b-blocker therapy would be
    indicated for his cardiac condition but is not a
    standard therapy for aneurysm.
92
Q
  1. A 70-year-old male underwent an open abdominal aortic aneurysm repair for ruptured
    aneurysm. He was stable during the procedure.
    In intensive care unit he was noted to have no
    urine output and was also noted to have large
    bloody bowel movement on first postoperative
    day. The next step for investigation includes:
    (A) Reexploration
    (B) Arterial blood gas evaluation for acidosis
    (C) CAT scan abdomen
    (D) Sigmoidscopy/colonoscopy
    (E) Antibiotics and hydration
A
  1. (D) Mortality associated with aortic aneurysm
    is usually around 0–3%. A ruptured AAA carries mortality in range of 60–80% depending on
    presentation. Risk of large-bowel ischemia with
    ruptured AAA is about 10%. The first investigation with patients where colonic ischemia is
    suspected is to perform sigmoidoscopy. All
    other investigations may be done but none of
    them would be the primary investigation for
    the suspected pathology.
93
Q
  1. A 69-year-old man was noted to have abdominal pain in left flank with severe hypotension
    and pulsatile mass in abdomen. He was taken
    to the operating room after he coded in the
    emergency room. Which of the following statements regarding ruptured abdominal aortic
    aneurysm is TRUE?
    (A) 10% of patient with ruptured aneurysm
    reach the Hospital.
    (B) Mortality is about 10%.
    (C) Aortic control is usually obtained by
    thoracotomy.
    (D) It cannot be treated by endovascular
    means.
    (E) Mortality following a code for ruptured
    AAA is 100%.
A
  1. (E) Ruptured AAA carries a mortality of
    40–50%. It is true that only 50% of all ruptured
    AAA reaches the hospital. Free peritoneal rupture carries a very high mortality. Thoracotomy
    is not the standard approach for proximal aortic
    control. Ruptured AAA can be treated with
    endovascular grafts. Preoperative hypotension
    is a good predictor of poor outcome but cardiac arrest is associated with 100% mortality in
    most of the studies.
    Answers and Explanations
94
Q
  1. A 82-year-old female presented with history of
    loss of vision in right eye for about 15 minutes
    and it cleared up. She has a history of diabetes
    and hypertension. She had which showed old
    infarct on right side. Carotid duplex showed
    that patient had 99% carotid artery stenosis.
    Which one of the following statements is
    TRUE?
    (A) 60% chance that extra cranial carotid
    artery stenosis is the cause of transient
    ischemic attack (TIA).
    (B) It is always due to platelet emboli.
    (C) 25% may be intracranial bleed.
    (D) 0.5 to 10% may have cardiac and other
    causes of TIA.
    (E) It is always due to thrombosis.
A
  1. (A) Neurological events are associated with
    extracranial carotid artery in about 60%. Fourty
    percent may have extracranial/intracranial
    cause for neurological events, which includes
    cardiac emboli, arch of aorta as source of emboli;
    intracranial bleed may be more than just a TIA.
    It is not always that platelet emboli are the cause
    of TIA, it could be due to atheroma. It is not
    always attributed to thrombus.
95
Q
  1. A 63-year-old male was noted to have a recent
    TIA. Patient was having recurrent episodes of
    TIA despite of being on aspirin and clopidogrel
    bisulfate. He does have a history of unstable
    angina. His workup includes magnetic resonance angiography (MRA) and carotid duplex.
    What are the appropriate treatment options?
    (A) Carotid endarterectomy for 50% carotid
    stenosis on MRA
    (B) Carotid endarterectomy for 60% stenosis
    on MRA without any treatment of
    unstable angina
    (C) Carotid endarterectomy for 90% stenosis
    with coronary artery bypass graft
    (CABG) at the same time
    (D) Start patient on heparin therapy and
    treat conservatively for carotid stenosis
    of 80%
    (E) Coronary angiogram with possible
    coronary intervention and simultaneous
    carotid angiogram and angioplasty and
    stenting
A
  1. (D) Asymptomatic carotid artery stenosis is only
    treated surgically if it is greater than 70% stenosis. The risk reduction with surgical treatment is
    favorable with 70% stenosis when compared
    to nonoperative treatment. Any symptomatic
    stenosis is an indication for surgical intervention
    including ulcerated plaque. Any amount of
    stenosis with unstable angina would need appropriate workup for cardiac risk prior to carotid
    intervention. Carotid endarterectomy and CABG
    are viable options if they are left main disease
    and have undergone coronary angiogram. In this
    patient the most appropriate treatment is option
    to perform coronary angiogram and possible
    carotid stenting if feasible. Role of anticoagulation to prevent recurrent TIA is not well established. Aspirin and clopidogrel bisulfate are
    appropriate options for TIA.
96
Q
  1. A 62-year-old man had right carotid endarterectomy 7 years ago. Now he has presented with
    80% stenosis on the same side. He has no symptoms from the stenosis. He has carotid artery
    stenosis on the opposite side of 80%. He does
    not have any history of TIA. What is the appropriate treatment for the patient?
    (A) Medical management with aspirin
    (B) Carotid artery redo surgery and patch
    angioplasty
    (C) Angiogram and angioplasty and stenting
    (D) Left carotid endarterectomy
    (E) Antiocoagulation of the patient to
    prevent stroke
A
  1. (D) Recurrent stenosis is secondary to intimal
    hyperplasia but it occurs in first two years. If
    more than two years, it is progression of disease and it does not carry high risk for
    embolization, so it is reasonable to observe it. It
    is also a surgery which carries higher stroke
    rate and morbidity with nerve injury which
    is in range of 7%. Patient is treated with
    antiplatelet therapy which includes aspirin and
    clopidogrel bisulfate. Anticoagulation with
    warfarin is not a standard therapy. It is appropriate to treat the opposite side with 80%
    carotid stenosis. Angiogram and angioplasty
    is an option but if the stenosis is significant
    and symptomatic. Priority in this case would be
    to treat the opposite side.
97
Q
  1. A 60-year-old male patient with bilateral carotid
    artery stenosis 90%, with history of right-sided
    weakness with resolution of symptoms in 15
    minutes. How would you treat the patient?
    (A) Right carotid endarterectomy
    (B) Left carotid endarterectomy
    (C) Right carotid angioplasty and stenting
    (D) Start patient on aspirin
    (E) Start patient on heparin
A
  1. (B) The treatment for symptomatic carotid
    artery stenosis greater than 70% is carotid
    endarterectomy. Since patient has left cerebral
    symptoms, it would be appropriate to treat that
    side first. Patient would need bilateral carotid
    endarterectomy but symptomatic side would
    be the first one to be operated. Heparin has no
    significant role in preventing stroke. Aspirin is
    a part of therapy but would not constitute a primary modality for treatment.
98
Q
10. A 72-year-old patient is noted to have neurological deficit following elective carotid
endarterectomy in recovery room. What is the
most appropriate treatment at this time?
(A) Carotid duplex
(B) CAT scan of brain
(C) Angiogram of cerebral vessels
(D) Heparin drip
(E) Exploration of the same side
A
  1. (E) In recovery room, the immediate approach
    would be to explore the patient. The cause for
    immediate stroke is usually technical and is
    most likely reversible if treated early on. All
    investigations are valid options once the technical cause is addressed and it would not be a
    primary option.
99
Q
  1. A 63-year-old man has had a cyanotic painful
    left fourth toe for 2 days. The dorsalis pedis
    and posterior tibial arteries are palpable on both
    sides. There is no history of cardiac or vascular
    disease. What is the most likely diagnosis?
    (A) Cardiac embolus
    (B) Atheroembolism
    (C) Lupus vasculitis
    (D) Digital atherosclerosis
    (E) Raynaud’s syndrome
A
  1. (B) All the listed conditions may result in
    isolated digital ischemia. In this age group,
    atheroembolism is the most likely diagnosis in a
    man. The atheroma is derived from an occult
    aortic aneurysm or a proximal ulcerative atherosclerotic lesion. This plaque or ulcer can be any
    part of the vascular tree proximal to the ischemic
    toe. Cardiac emboli also are common in this
    age group but are a less likely cause in the
    absence of previous MI, arrhythmia, or valvular disease.
100
Q
  1. A 40-year-old chronic smoker presents with
    ulceration of the tip of the right second, third,
    and fourth toes. He gives a history of recurrent migratory superficial phlebitis of the feet
    occurring a few years ago. Physical examination findings are remarkable for absent bilateral
    posterior tibial and dorsalis pedis pulses with
    palpable popliteal pulses. What is the single
    most important step in management?
    (A) Multiple toe amputations
    (B) Long-term anticoagulant therapy
    (C) Immediate operative intervention
    (D) Angiography followed by bypass surgery
    (E) Cessation of smoking
A
  1. (E) This patient suffers from thromboangiitis
    obliterans (Buerger’s disease), a disease found
    most frequently in white men between 20 and
    40 years of age. It is a form of panvasculitis
    involving the artery, vein, and nerve. Heavy
    tobacco smoking is strongly associated with
    this disease. Early in the course of the disease,
    there is involvement of the superficial veins,
    producing recurrent migratory superficial
    phlebitis. The distribution of arterial involvement is usually segmental, involving the
    peripheral arteries. In the lower extremities,
    the disease occurs generally beyond the
    popliteal arteries and distal to the forearm in
    the upper extremities. As long as ulceration or
    gangrene is confined to a digit, amputation
    should be postponed as long as possible unless
    rest pain or infection cannot be otherwise controlled. Bypass surgery is rarely indicated, and
    long-term anticoagulation has not been of
    much benefit. The most important aspect of
    treatment is cessation of smoking, which can
    halt progression of the disease.
    228 10: Vascular
101
Q
  1. A middle-aged man is found to have a small
    pulsating mass at the level of the umbilicus
    during a routine abdominal examination. What
    is the best initial test to establish the diagnosis?
    (A) Aortography
    (B) Ultrasound
    (C) Computed tomography (CT)
    (D) Magnetic resonance imaging (MRI)
    (E) Plain films of the abdomen
A
  1. (B) Although aortography, CT, and MRI can
    all establish the diagnosis of abdominal aortic
    aneurysm, ultrasound remains the best screening test. It is the preferred method for making
    the initial diagnosis, because it is reliable, inexpensive, and noninvasive. Aortography is used
    infrequently because of the small but definite
    risk it entails and because diagnosis can be
    made by other means. Once the aneurysm
    meets the criteria for repair, then a CT scan is
    done preoperatively to establish the true size
    and to delineate the aneurysm more accurately.
    Plain films of the abdomen are inaccurate in
    establishing the diagnosis.
102
Q
  1. A 58-year-old woman is found to have a right
    carotid bruit on routine examination. She is
    completely asymptomatic. A carotid duplex
    scan and carotid arteriogram (Fig. 10–1) reveal
    a right carotid stenosis. Which of the following
    statements is true?
    (A) Operative treatment is indicated if the
    stenosis is greater than 80%, even if the
    patient is asymptomatic.
    (B) The incidence of stroke can be decreased
    by prophylactic carotid endarterectomy
    in patients with as little as 40% stenosis.
    (C) Aspirin is always a superior treatment
    to surgery regardless of the degree of
    stenosis.
    (D) If symptoms eventually develop, they
    are invariably TIAs, not stroke.
    (E) Neither surgery nor aspirin is indicated,
    because the patient is asymptomatic.
A
  1. (A) Operative treatment is indicated if the diameter of the stenosis is greater than 60%, even if
    the patient is asymptomatic. The value of prophylactic carotid endarterectomy, for hemodynamically significant carotid stenosis, decreases
    the incidence of subsequent cerebral ischemic
    events if performed with morbidity and mortality rates under 4%. Several studies including
    asymptomatic carotid artery surgery (ACAS)
    have shown that surgical treatment is superior
    to medical management if the stenosis is 60% or
    greater. The ACAS trial has shown the benefits
    of surgical treatment over medical management
    if the stenosis is greater than 60%. However,
    there are no data to support the use of carotid
    endarterectomy in asymptomatic patients with
    stenosis of less than 60%. If ischemic events
    eventually develop, stroke can be the presenting
    symptom.
103
Q
  1. A 57-year-old male smoker is referred to you
    because of two episodes of right upper extremity weakness over the past 6 months, each lasting for 10–15 minutes. Findings on CT scan of
    the head are negative. An angiogram shows a
    75% stenosis of the left carotid artery. What is
    the most appropriate treatment?
    (A) Antiplatelet therapy
    (B) Oral anticoagulants
    (C) Carotid endarterectomy
    (D) Carotid artery bypass to vertebral system
    (E) Surgery only if a stroke develops
A
  1. (C) This patient is experiencing recurrent left
    hemispheric TIA with a hemodynamically significant stenosis of the left carotid artery. This
    is clearly an indication for surgery because
    operative management is superior to aspirin
    in symptomatic carotid bifurcation disease with
    stenosis greater than 70%. Oral anticoagulants
    may decrease the incidence of TIAs but not of
    completed strokes, and they are associated with
    a considerable risk of hemorrhage. Carotid
    endarterectomy, and not carotid artery bypass,
    is the surgical procedure of choice. Surgical
    treatment must be performed before and not
    after major neurologic deficits are produced
    from cerebral infarction.
104
Q
  1. A 24-year-old man complains of progressive
    intermittent claudication of the left leg. On
    examination, the popliteal, dorsalis pedis, and
    posterior tibial pulses are normal; but they disappear on dorsiflexion of the foot. What is the
    most likely diagnosis?
    (A) Embolic occlusion
    (B) Thromboangiitis obliterans
    (C) Atherosclerosis obliterans
    (D) Popliteal artery entrapment syndrome
    (E) Cystic degeneration of the popliteal
    artery
A
  1. (D) Popliteal artery entrapment syndrome consists of intermittent claudication caused by an
    abnormal relation of that artery to the muscles,
    usually the medial head of the gastrocnemius
    muscle. As a consequence of developmental
    abnormalities, the popliteal artery may be compressed by the medial head of the gastrocnemius muscle, resulting in ischemia of the leg at
    an unusually early age. On examination, the
    pulses may be diminished or absent, but they
    may also be normal and be made to disappear
    on dorsiflexion of the foot. Angiography is
    essential to establish the diagnosis.
105
Q
  1. Four days after undergoing hysterectomy, a 30-
    year-old woman develops phlegmasia cerulea
    dolens over the right lower extremity. What is
    the most appropriate treatment?
    (A) Bed rest and elevation
    (B) Systemic heparinization
    (C) Venous thrombectomy
    (D) Prophylactic vena caval filter
    (E) Local urokinase infusion
A
  1. (C) Phlegmasia cerulae (blue) dolens, indicates
    that major venous obstruction has occurred.
    The standard treatment for postoperative
    thrombosis includes bed rest and anticoagulation. Venous thrombectomy may be indicated
    when impending gangrene is noted. Vena caval
    filters are inserted in patients with established pulmonary emboli, but they may be
    considered as a prophylactic measure when
    iliofemoral thrombosis is massive. They are
    also inserted as an adjunct to venous thrombectomy along with creation of an arteriovenous
    fistula to prevent the venous system from
    rethrombosing. Thrombolysis of major venous
    thrombi requires placement of a multihole
    pigtail catheter inside the thrombus and
    administration of tPA, including systemic
    heparinization and is therefore contraindicated
    postoperatively.
106
Q
  1. A 21-year-old woman is referred to your office
    because of multiple lower extremity varicose
    veins. She has large varicosities in the distribution of the long saphenous vein. What is the
    next step in management?
    (A) A ligation and stripping operation
    (B) Ligation of both the long and short
    saphenous system
    (C) Sclerotherapy
    (D) Duplex evaluation along with clinical
    correlation as an essential initial step
    (E) Compression stockings and
    anticoagulation therapy
A
  1. (D) A through clinical evaluation followed by
    a venous duplex examination are the two most
    important steps in managing varicose vein of
    the lower extremity. An asymptomatic patient
    without complications of phlebitis, ulceration,
    or hemorrhage should be treated with compression stocking. Duplex evaluation will help
    map the valvular incompetence of the superficial and deep system including the perforators
    that guide the extent of the initial surgical intervention, and also investigate if these are primary or secondary varicosities. Sclerotherapy is
    an alternative to surgery but in the presence of
    saphenofemoral, saphenopopliteal, or perforator reflux is associated with a high incidence of
    recurrence and complications.
    Answers: 6–18 229
107
Q
  1. A 45-year-old woman undergoes cardiac
    catheterization through a right femoral approach.
    Two months later, she complains of right lower
    extremity swelling and notes the appearance of
    multiple varicosities. On examination, a bruit is
    heard over the right groin. What is the most
    likely diagnosis?
    (A) Femoral artery thrombosis
    (B) Superficial venous insufficiency
    (C) Arteriovenous (AV) fistula
    (D) Pseudoaneurysm
    (E) Deep vein insufficiency
A
  1. (C) A traumatic AV fistula results from a penetrating injury to adjacent artery and vein, permitting blood flow from the injured artery
    into the vein. The iatrogenic injury in this
    case occurred during cardiac catheterization.
    Femoral artery thrombosis results in signs of
    limb ischemia. A bruit is usually not heard
    with venous insufficiency. Traumatic pseudoaneurysm presents as an enlarging pulsating
    mass. Once the diagnosis of AV fistula is made,
    an angiogram is performed, and surgical repair
    (division of the fistula and reconstruction of
    the artery and preferably of the injured vein as
    well) is carried out.
108
Q
  1. A young basketball player develops an acute
    onset of subclavian vein thrombosis (effort
    thrombosis) after heavy exercise. What is the
    next step in management?
    (A) Active exercise of the limb
    (B) Anti-inflammatory drugs
    (C) Thrombolytic therapy
    (D) Antibiotics
    (E) First-rib resection
A
  1. (C) Effort thrombosis, also called Paget-von
    Schroetter syndrome, is the development of
    thrombosis of the axillary-subclavian vein as a
    result of injury or compression. It occurs primarily in young athletes and is disabling.
    When these patients are seen early, thrombolytic therapy is the first step in management
    and is followed by a venogram to detect correctable lesions. If effort thrombosis is associated with thoracic outlet syndrome, then
    thrombolytic therapy should be followed by
    cervical rib resection. If the condition is chronic,
    thrombolytic therapy might not be successful;
    these patients usually respond to limb elevation and anticoagulation.
109
Q
  1. A middle-aged man undergoes a left belowknee amputation for left-foot gangrene secondary to arterial occlusive disease. Which of
    the following statements is true after the belowknee amputation?
    (A) There is less efficient function than after
    a through-knee amputation.
    (B) Stump prognosis can be judged by
    transcutaneous oxygen monitoring.
    (C) Poor prognosis is inevitable if Doppler
    fails to record a pulse at that level.
    (D) The fibula and tibia are of equal length.
    (E) The level of transection is 5 cm above
    the medial malleolus.
A
  1. (B) Stump prognosis can be judged by transcutaneous oxygen monitoring. Doppler is not
    fully reliable to select the level of transection,
    because it cannot calculate the quantity of vascular flow. Transcutaneous oxygen (PO2 >40
    mm Hg) offers a fairly accurate prediction of a
    favorable result; although, Doppler fails to confirm a patient pulse at the level of transection.
    On the other hand, a duplex evaluation with
    blood flow of more than 50 cm/s is also a fairly
    accurate predictor for stump prognosis. The
    level of transection is 13–15 cm below the level
    of the medial condyle of the tibia.
110
Q
22. A 72-year-old retired banker complains of leftleg intermittent claudication while playing golf.
An angiogram shows occlusion of the superficial femoral artery and reconstitution of the
popliteal artery below the knee. What is the
treatment of choice?
(A) A vigorous exercise program
(B) Endarterectomy of the superficial
femoral artery
(C) Femoropopliteal bypass with expanded
polytetrofluoroethylene (PTFE) graft
(D) In situ femoropopliteal bypass
(E) Femoropopliteal bypass with reversed
saphenous vein graft
A
  1. (A) If claudication is the only symptom, elective vascular reconstruction is considered only if
    claudication is disabling and interferes with dayto-day activity. Because the risk of gangrene,
    occurring in a patient who has only claudication,
    is small, this alone does not constitute a clear-cut
    indication for operation. Vigorous exercise programs have resulted in marked improvement
    in claudicants. Revascularization surgery is usually reserved for rest pain or tissue loss (nonhealing ulcer, gangrene). Addition of a
    phosphodiastraze inhibitor, cilostazol (pletal),
    or pentoxiphyline (trental) can help increase the
    claudication distance. It should also be kept in
    mind that an angiogram is not indicated for
    claudication. An initial evaluation with noninvasive vascular studies is the investigation of
    choice. Angiogram is only requested if the decision is made to intervene surgically.
111
Q
  1. A 40-year-old patient undergoes a CT scan of
    the abdomen for nonspecific abdominal pain. A
    splenic artery aneurysm is incidentally identified. What is true of the splenic artery aneurysm?
    (A) It requires splenectomy for optimal
    treatment.
    (B) It is more common in men.
    (C) It is caused by atherosclerosis in most
    cases.
    (D) It may rupture during pregnancy.
    (E) It is rarely calcified on an abdominal
    x-ray.
A
  1. (D) Splenic artery aneurysms are rare and are
    most frequently caused by medial necrosis.
    Small asymptomatic aneurysms caused by atherosclerosis are more commonly incidental
    findings at autopsy. Larger (>3 cm) aneurysms
    predominate in women and characteristically
    may rupture during late pregnancy. Rupture
    may be preceded by an initial warning bleed
    into the retroperitoneum, with massive bleeding following after 1 or 2 days.
112
Q
  1. A 70-year-old man with a long-standing history
    of diabetes develops gangrene of the right
    second toe. What is true of his diabetic foot?
    (A) Dorsalis pedis and posterior tibial
    arteries are always absent.
    (B) Gangrene of the toe always requires
    urgent below-knee amputation.
    (C) Arterial reconstruction is invariably
    required.
    (D) His right femoral artery is most probably occluded or stenosed.
    (E) Trophic ulcers are sharply demarcated.
A
  1. (E) Patients with a diabetic foot may have localized arterial occlusion involving the popliteal
    artery and its branches, usually sparing the
    femoral artery. Although patients have gangrene
    of the toes, there may be a palpable pulse in the
    foot. In the presence of localized disease, trophic
    ulcers and even gangrene of the toes may
    respond to local foot care, and major vascular
    reconstruction or amputations are not required.
    The trophic ulcers have punched sides. Patients
    may not realize the gravity of localized gangrene with spreading cellulitis, which develops
    because of the neurotropic nature of the lesions
    with the absence of pain sensation.
113
Q
  1. Eleven years after undergoing right modified
    radical mastectomy, a 61-year-old woman
    develops raised red and purple nodules over
    the right arm. What is the most likely diagnosis?
    (A) Lymphangitis
    (B) Lymphedema
    (C) Lymphangiosarcoma
    (D) Hyperkeratosis
    (E) Metastatic breast cancer
A
  1. (C) Lymphangiosarcoma is a rare complication of
    long-standing lymphedema, most frequently
    described in a patient who has previously undergone radical mastectomy (Stewart-Treves syndrome). It usually presents as blue, red, or purple
    nodules with satellite lesions. Early metastasis,
    mainly to the lung, may develop if it is not recognized early and widely excised. Lymphedema
    is a complication of radical mastectomy and presents as diffuse swelling and nonpitting edema of the limb. Lymphangitis and hyperkeratosis are complications of lymphedema.
114
Q
  1. Four days after undergoing subtotal gastrectomy for stomach cancer, a 58-year-old woman
    complains of right leg and thigh pain, swelling
    and redness, and has tenderness on examination. The diagnosis of deep vein thrombosis is
    entertained. What is the initial test to establish
    the diagnosis?
    (A) Venography
    (B) Venous duplex ultrasound
    (C) Impedance plethysmography
    (D) Radio-labeled fibrinogen
    (E) Assay of fibrin/fibrinogen products
A
  1. (B) The most accurate method of confirming the
    diagnosis of venous thrombosis is the injection
    of contrast material to visualize the venous
    system (venography). However, this method is
    invasive and time-consuming and must be done
    in the radiology suite. Venous duplex ultrasound is noninvasive, can be done bedside, and
    has a sensitivity and specificity of 96 and 100%,
    respectively. The other methods listed are used
    less often in certain selected patients.
115
Q
  1. A middle-age woman has right leg and foot
    nonpitting edema associated with dermatitis
    and hyperpigmentation. The diagnosis of
    chronic venous insufficiency is made. What is
    the treatment of choice?
    (A) Vein stripping
    (B) Pressure-gradient stockings
    (C) Skin grafting
    (D) Perforator vein ligation
    (E) Valvuloplasty
A
  1. (B) The mainstay of treatment of chronic venous
    insufficiency and its complication, venous stasis
    ulceration, is conservative management. Elastic
    stocking support, frequent elevation of the legs,
    and avoidance of prolonged sitting and standing is used for venous insufficiency in the
    absence of ulceration. If venous stasis ulcers
    develop, then paste boots (e.g., Unna’s boots)
    are used along with appropriate bed rest and
    foot elevation until the ulcer heals. Patients
    whose ulcers fail to heal after such conservative
    management may need perforator vein ligation. Skin grafting should be considered for
    chronic stasis ulcers that are large, and perforator incompetance has been treated. Venous
    reconstruction procedures, including valvuloplasty, can be useful for a selected group of
    patients, especially those with venous claudication to less than half a block, that have been
    treated with all the procedures above, including
    stripping and ligation. Unlike previous opinions, superficial venous stripping and ligation
    is not always contraindicated in the presence of
    chronic venous insufficiency and even previous history of deep vein thrombosis.
116
Q
  1. A 55-year-old woman has bilateral leg edema
    associated with thick, darkly pigmented skin. A
    Trendelenburg’s test is done, and results are
    interpreted as positive/positive. What does
    this patient have?
    (A) Competent varicose veins/competent
    perforators
    (B) Competent varicose veins/incompetent
    perforators
    (C) Deep vein thrombosis (DVT)
    (D) Incompetent varicose veins/competent
    perforators
    (E) Incompetent varicose veins/incompetent
    perforators
A
  1. (E) The Trendelenburg’s test is a two-part test
    used to access the competency of the superficial
    and perforating veins. The legs are elevated to
    evacuate the veins, and pressure is applied to
    the saphenofemoral junction either by hand or
    tourniquet. The four possible results are: (a) negative/negative response if there is gradual filling
    of veins from below and continued slow filling
    after release of pressure, indicating absence of
    incompetent superficial and perforating veins;
    (b) negative/positive response if there is gradual
    filling of veins from below while there is rapid
    retrograde filling after release of pressure, indicating incompetent superficial veins only; (c)
    positive/negative response if there is rapid initial filling of the veins from below while only
    continued slow filling after the release of pressure, indicating incompetent perforators only;
    and (d) positive/positive response if there is
    rapid filling of the saphenous vein before and
    after release of pressure, indicating incompetent
    superficial and perforating veins.
117
Q
  1. A middle-aged man known to have peptic
    ulcer disease is admitted with upper gastrointestinal (GI) bleeding. During his hospital stay,
    he develops DVT of the left lower extremity.
    What is the most appropriate management?
    (A) Anticoagulation
    (B) Observation
    (C) Thrombolytic therapy
    (D) Inferior vena cava (IVC) filter
    (E) Venous thrombectomy
A
  1. (D) The main treatment of DVT is adequate anticoagulation. However, if pulmonary embolism
    develops during anticoagulant therapy or if there
    is contraindication to anticoagulation, the insertion of an IVC filter is indicated either to prevent
    occurrence of or to offer prophylaxis against
    recurrence of pulmonary embolism (Fig. 10–4).
    Observation alone leaves the patient unprotected
    against pulmonary embolism, and operative
    thrombectomy is reserved for limb salvage in
    the presence of impending venous gangrene.
    Obviously, if anticoagulation is contraindicated
    (as in the patient presented), thrombolytic therapy cannot be used.
118
Q
  1. A 70-year-old executive is complaining of
    three-block intermittent claudication of both
    legs. What is the percentage chance of his
    developing limb-threatening gangrene?
    (A) Less than 10%
    (B) 20%
    (C) 45%
    (D) 60%
    (E) More than 75%
A
  1. (A) The relatively benign course of intermittent claudication has been well established. The
    risk of gangrene developing within 5 years in
    an extremity with claudication as the only
    symptom is only about 5%. The patient must be
    encouraged to stop smoking, to exercise, and be
    placed on a diet that lowers cholesterol.
119
Q
31. Thirty-six hours after undergoing an abdominal aortic aneurysm repair, a 70-year-old
woman develops abdominal distension associated with bloody diarrhea. What is the most
likely diagnosis?
(A) Aortoduodenal fistulas
(B) Diverticulitis
(C) Pseudomembranous enterocolitis
(D) Ischemic colitis
(E) Acute hepatic failure
A
  1. (D) The occurrence of bowel movements
    during the first 24–72 hours after repair of an
    abdominal aortic aneurysm (especially if the
    hemoccult test is positive), should raise suspicion for ischemic colitis. It may develop as a
    result of interruption of flow to the inferior
    mesenteric artery with inadequate collateral
    circulation from either the superior mesenteric
    artery or the iliac arteries. Aortoduodenal fistula is a late complication of aneurysm repair.
    Pseudomembranous enterocolitis occurs late
    in the postoperative course.
120
Q
32. A 65-year-old man is referred to you because of
an incidental finding of a 3-cm left popliteal
aneurysm (Fig. 10–2). The patient is completely
asymptomatic and has normal pulses. How
should the aneurysm be treated?
(A) It should be observed.
(B) It should be repaired because it may
lead to spontaneous rupture.
(C) It should be repaired only if it is larger
than 5 cm.
(D) It should be repaired because of its
tendency to either undergo thrombosis
or embolize distally.
(E) It should be repaired because of its
tendency to cause nerve compression if
it enlarges.
A
  1. (D) Popliteal aneurysms are usually arteriosclerotic and are bilateral in at least 50% of
    cases. Any popliteal aneurysm twice the size of
    the normal artery is an indication for surgical
    repair. Although often asymptomatic and
    small, they should be treated surgically because
    of their propensity to produce limb-threatening
    ischemia related to thrombosis or embolism.
    Spontaneous rupture and/or nerve compression are rare complications of a popliteal
    aneurysm. The ideal repair consists of ligation
    of the aneurysm, including its branches and a
    bypass to the open distal vessels.
121
Q
  1. A 72-year-old woman falls at home after an
    episode of dizziness. She had been complaining
    of low-back pain for 3 days before the fall. In the
    emergency department, she is hypotensive and
    has cold, clammy extremities. A pulsating
    mass is palpable on abdominal examination.
    Following resuscitation, the next step in the management should involve which of the following?
    (A) Peritoneal lavage
    (B) Immediate abdominal exploration
    (C) CT scan of the abdomen
    (D) Abdominal aortogram
    (E) Abdominal ultrasound
A
  1. (B) The presence of acute vascular collapse with
    history of abdominal or flank pain and associated pulsating abdominal mass is characteristic
    of a ruptured abdominal aneurysm. Operation
    should be performed as quickly as possible,
    because the first priority is to control the hemorrhage. No time should be lost in obtaining
    diagnostic studies, because these patients often
    crash in the radiology suite. These patients
    should not be resuscitated aggressively, because
    an increase in systolic pressure will only cause
    more intra-abdominal hemorrhage.
122
Q
34. A 60-year-old man complains of dizziness, vertigo, and mild right-arm claudication. On physical examination, there is decreased pulse and
blood pressure of the right upper extremity.
What is the treatment of choice?
(A) Anticoagulation
(B) Repair of coarctation of the aorta
(C) Ligation of vertebral artery
(D) Carotid endarterectomy
(E) Carotid subclavian bypass
A
  1. (E) The clinical picture presented is that of a subclavian artery stenosis resulting in subclavian
    steal syndrome, represented by vertebrobasilar
    symptoms and extremity ischemia. The symptoms are due to a decrease of posterior circulation
    (vertebral artery) blood flow. Claudication occurs
    more commonly than ischemic findings. Most
    patients have no triggering events, and the
    symptoms are not readily reproducible. Carotid
    subclavian bypass restores the circulation
    beyond the stenotic area and corrects the steal
    syndrome. Ligation of the vertebral artery will
    correct the steal syndrome but will not improve
    the circulation of the arm. Anticoagulation has no
    role in the treatment of this entity. Other treatment options include subclavian artery transposition, axilloaxillary bypass, and subclavian
    artery angioplasty. Coarctation of the aorta
    results in pulse and pressure difference between
    the upper and lower extremities.
123
Q
  1. An 18-year-old man develops a painful, swollen
    leg while training for the New York Marathon.
    There is tenderness in the calf and ecchymosis
    is present. What is the most likely diagnosis?
    (A) Cellulitis
    (B) DVT
    (C) Superficial thrombophlebitis
    (D) Tear of the plantaris muscle
    (E) Medical lemniscus tear
A
  1. (D) Spontaneous thrombophlebitis in this age
    group is unlikely. Plantaris or gastrocnernius
    tear may occur during physical exertion involving running or walking, causing a sharp pain in
    this region. After resolution of a hematoma in
    this region, it may be difficult to exclude cellulitis if there is any question that the integrity
    of the skin has been damaged. In superficial
    thrombophlebitis, there is tenderness along the
    distribution of the long or short saphenous
    veins. A tear of the medial lemniscus of the knee
    joint is detected by tenderness over the medical
    aspect of the knee joint during flexion and internal rotation of the knee joint (McMurray sign).
124
Q

Four days after suffering MI, a 78-year-old woman
suddenly develops severe diffuse abdominal pain.
Her electrocardiogram (ECG) shows atrial fibrillation. On examination, the abdomen is soft, minimally
tender, and slightly distended. Hyperactive bowel
sounds are present.

36. What is the most likely diagnosis?
(A) Mesenteric embolus
(B) Nonocclusive ischemic disease
(C) Perforated peptic ulcer
(D) Congestive heart failure (CHF)
(E) Digoxin toxicity
A
  1. (A) Patients with atrial fibrillation are more
    likely to develop emboli to different sites
    throughout the body. Nonocclusive ischemic
    disease is characterized by spasm of the major
    mesenteric arterial vessels, with a characteristic beading effect. Early recognition may result
    in improvement with direct intra-arterial infusion of papaverine (which causes vasodilation),
    thus avoiding operative intervention.
125
Q

Four days after suffering MI, a 78-year-old woman
suddenly develops severe diffuse abdominal pain.
Her electrocardiogram (ECG) shows atrial fibrillation. On examination, the abdomen is soft, minimally
tender, and slightly distended. Hyperactive bowel
sounds are present.
37. The most appropriate initial examination consists of which of the following?
(A) Gastrografin upper GI series
(B) White blood cell (WBC) counts and serial
abdominal examination
(C) Colonoscopy
(D) Diagnostic peritoneal lavage
(E) Angiography

A
  1. (E) Clinical findings of peritoneal irritation and
    leukocytosis in patients with suspected visceral
    ischemia indicate necrosis of ischemic bowel.
    Immediate arteriography is required to establish
    the diagnosis and initiate treatment to restore
    circulation before massive bowel infarction,
    232 10: Vascular
    acidosis, and possible perforation occur. The
    most likely diagnosis is a mesenteric embolus
    arising from the heart, especially in the presence of atrial fibrillation. The catheter should be
    left in place to allow papaverine infusion to an
    area of borderline ischemic bowel.
126
Q
  1. A 28-year-old woman has new-onset hypertension and a bruit on abdominal examination.
    An arteriogram shows fibromuscular dysplasia
    (FMD) of the right renal artery. What is the best
    treatment option?
    (A) Aortorenal saphenous vein bypass
    (B) Patch angioplasty of the renal artery
    (C) Percutaneous transluminal angioplasty
    (PTA)
    (D) Transaortic renal endarterectomy
    (E) Hepatorenal bypass
A
  1. (C) Among all causes of renovascular hypertension, FMD responds best to angioplasty.
    Intermediate results of PTA for FMD are similar
    to those of bypass. PTA has lower morbidity,
    causes less discomfort, and is less expensive.
    Recurrence can be treated by repeated PTA.
127
Q

A 60-year-old man with a history of atrial fibrillation is found to have a cyanotic, cold right lower
extremity.

39. The embolus is most probably originating from
which of the following?
(A) An atherosclerotic plaque
(B) An abdominal aortic aneurysm
(C) Heart
(D) Lungs
(E) Paradoxical embolus
A
  1. (C) The heart is the origin of about 90% of
    lower extremity emboli. The causes are usually mitral stenosis, atrial fibrillation, or MI. A
    rare source of left atrial emboli is a left atrial
    myxoma. The remaining 10% arise from ulcerated plaques in the aorta or peripheral arteries.
    Paradoxical emboli arising from the venous
    system may reach the arterial circulation through
    a patent foramen ovale.
128
Q

A 60-year-old man with a history of atrial fibrillation is found to have a cyanotic, cold right lower
extremity.

40. Which is the most common site at which an
arterial embolus lodges?
(A) Aortic bifurcation
(B) Popliteal artery
(C) Tibial arteries
(D) Common femoral artery
(E) Iliac artery
A
  1. (D) Arterial emboli usually lodge proximal to
    bifurcations, the most common site being the
    common femoral artery.
129
Q

A 60-year-old man with a history of atrial fibrillation is found to have a cyanotic, cold right lower
extremity.

41. What is the most appropriate management?
(A) Embolectomy
(B) Lumbar sympathectomy
(C) Bypass surgery
(D) Amputation
(E) Arteriography
A
  1. (A) Once the diagnosis is made clinically,
    heparin is administered intravenously to prevent the development of thrombi distal to
    the embolus. Then embolectomy can be done
    in most instances under local anesthesia.
    Arteriography to confirm what is already clinically apparent only delays the needed surgical
    procedure. If there is a doubt, duplex evaluation will help confirm the diagnosis. Lumbar
    sympathectomy locks are of dubious value. In
    patients who have known occlusive disease,
    absent pulses in the contralateral extremity,
    absence of clinical features of hyperacute
    ischemia would be best managed by an angiogram and thrombolytic infusion.
130
Q
  1. An elderly patient with ischemic rest pain is
    found to have combined aortoiliac and
    femoropopliteal occlusive disease. What is the
    treatment of choice?
    (A) Aortofemoral bypass
    (B) Femoropopliteal bypass
    (C) Aortofemoral and femoropopliteal bypass
    (D) Lumbar sympathectomy
    (E) Vasodilator therapy
A
  1. (A) Patients with combined segmental occlusive disease require correction of proximal
    hemodynamically significant disease before
    distal (infrainguinal) bypass. Only about 20%
    of patients undergoing aortofemoral reconstruction in the presence of superficial femoral
    artery occlusion will subsequently require
    femoropopliteal bypass. Combined procedures
    should be reserved for patients with severe lifethreatening ischemia. Lumbar sympathectomy
    and vasodilator therapy are ineffective in treating severe arterial occlusive disease.
131
Q
  1. A 66-year-old woman has a 5.5-cm infrarenal
    abdominal aortic aneurysm. What is the most
    common manifestation of such an aneurysm?
    (A) Abdominal or back pain
    (B) Acute leak or rupture
    (C) Incidental finding on abdominal
    examination
    (D) Atheroembolism
    (E) Spontaneous thrombosis
A
  1. (C) Most patients are unaware of their abdominal
    aneurysm until it is incidentally discovered by
    their physician. The importance of careful deep
    palpation of the abdomen cannot be overemphasized. On occasion, these aneurysms may
    expand, causing abdominal or back pain, and
    may even leak or rupture, mimicking other acute
    intra-abdominal conditions. Signs and symptoms
    of acute ischemia in the lower extremities are rare
    and usually follow thrombosis or embolization
    from an abdominal aneurysm.
132
Q
  1. A 72-year-old man complains of bilateral thigh
    and buttock claudication of several months
    duration. He was told by his physician that the
    angiogram revealed findings indicating that he
    has Leriche syndrome. What does this patient
    have?
    (A) Abdominal aortic aneurysm
    (B) Aortoiliac occlusive disease
    (C) Iliac artery aneurysm
    (D) Femoropopliteal occlusive disease
    (E) Tibial occlusive disease
A
  1. (B) Leriche syndrome consists of the manifestations of aortoiliac occlusive disease and includes
    thigh and buttock claudication, atrophy of the
    leg muscles, diminished femoral pulses, and
    impotence in men.
133
Q
  1. A young woman develops a left femoral arteriovenous fistula a few months after a stab
    wound to the groin. Which of the following
    physiological changes (Nicoladoni-Branham
    sign) is elicited on physical examination?
    (A) Appearance of CHF when the artery
    proximal to the fistula is compressed
    (B) Slowing of the pulse rate when the
    fistula is compressed
    (C) A rise in the pulse rate when the artery
    distal to the fistula is compressed
    (D) A bruit heard only after the fistula is
    occluded
    (E) Absent dorsalis pedis after leg is elevated
A
  1. (B) The Nicoladoni-Branham sign can be elicited
    in some patients with an AV fistula. Occlusion of
    the fistula or the artery proximal to the fistula
    may result in slowing of the heart rate. By
    this compression, the peripheral resistance is
    increased, venous return is decreased, and the
    pulse rate falls.
134
Q
  1. A young patient sustains blunt trauma to his
    right knee that results in acute thrombosis of
    his popliteal artery. Which tissue is most sensitive to ischemia?
    (A) Muscle
    (B) Nerve
    (C) Skin
    (D) Fat
    (E) Bone
A
  1. (B) Peripheral nerve endings are the tissues most
    sensitive to anoxia in the extremity. Therefore,
    paralysis and paresthesia are most important
    when evaluating an extremity with acute arterial
    occlusion. The second most sensitive tissue is
    the muscle. This is why an extremity with paralysis and paresthesia will develop gangrene if
    circulation is not restored. Gangrene is less likely
    to occur if signs of ischemia are present, but
    motor and sensory functions are intact.
135
Q
  1. Seven years after undergoing resection of an
    abdominal aortic aneurysm and repair with a
    Dacron graft, a 65-year-old man develops an
    aortoenteric fistula. What would be the safest
    method to treat this patient?
    (A) Administration of a prolonged course of
    antibiotics
    (B) Removal of the Dacron graft, closure of
    the enteric defect, and the insertion of a
    new aortic graft
    (C) Closure of the enteric fistula, removal of
    the Dacron graft, ligation of the infrarenal
    aorta, and insertion of an extra-anatomic
    axillobifemoral bypass graft
    (D) Division of the fistula, closure of the
    aortic and enteric defects, and
    interposition of omentum in between
    (E) Closure of the enteric fistula, removal of
    the Dacron graft, ligation of the infrarenal
    aorta, and insertion of an extra-anatomic
    bypass at a later date
A
  1. (E) The use of an extra-anatomic bypass (axillobifemoral) is indicated in the presence of
    “hostile” abdomen (infection, dense and severe
    adhesions, tumors) or if the patient is too sick to
    undergo an abdominal operation. If a previously placed graft is contaminated (infection,
    Answers: 30–47 233
    aortoenteric fistula), the graft must be removed,
    and the enteric defect must be closed. Although
    some surgeons advocate removing the infected
    graft and replacing it in situ with a new graft, the
    safest approach remains the extra-anatomic
    route to restore circulation to the lower extremities (axillobifemoral bypass).
136
Q
  1. A 24-year-old male cyclist undergoes repair of
    both popliteal artery and vein following a gunshot wound to the right knee. Thirty-six hours
    postoperatively, there is increasing swelling of
    the leg and foot, and the patient complains of
    increasing foot pain and inability to move his
    toes. His pedal pulses are palpable. What is the
    most immediate next step that should be
    undertaken?
    (A) Arteriography
    (B) Leg and foot elevation
    (C) Fasciotomy
    (D) Venography
    (E) Immediate reexploration of the popliteal
    space
A
  1. (C) Compartment syndrome can occur following repair of vascular injuries, especially if
    ischemia time is more than 6 hours or if there
    have been substantial periods of shock. Other
    instances include the combination of arterial
    and venous injury and the presence of concomitant soft-tissue crush injury or bone fracture. Compartment swelling and tenderness,
    pain disproportionate to the physical findings,
    paresthesia, and weakness are all clinical signs
    of compartment syndrome and require urgent
    surgical decompression. A palpable pulse does
    not rule out the presence of a compartment
    syndrome, because compartment pressures are
    high, even before loss of a palpable pulse.
137
Q
  1. A homeless elderly man is brought to the emergency department after sustaining frostbite to
    both feet. What is the most appropriate immediate management?
    (A) Slow rewarming at room temperature
    (B) Amputation of the gangrenous toes
    (C) Rapid rewarming with warm water
    (D) Rapid rewarming with hot water or dry
    heat
    (E) Thorough debridement of blisters and
    devitalized tissue
A
  1. (C) Rapid warming of the injured tissue is the
    most important aspect of treatment. The frozen
    tissue should be placed in warm water, with a
    temperature in the range of 408–448ºC. Dry
    heat or hot water carries the risk of thermal
    injury because of decreased sensation in the
    injured part. Opening of blisters and debridement of devitalized tissue are contraindicated.
    Demarcation of gangrenous areas should be
    carefully observed, often for several weeks,
    before amputation is performed. The extremity
    should be elevated, tetanus prophylaxis should
    be administered as indicated, and antibiotics
    should be given in the presence of open wounds.
138
Q
  1. A 55-year-old woman who comes from a highaltitude location is diagnosed as having a
    carotid body tumor (Fig. 10–3). What is true of
    these tumors?
    (A) They most frequently present as a
    painless neck mass.
    (B) They arise from endothelial cells.
    (C) They are usually hypovascular.
    (D) They frequently manifest with a stroke.
    (E) They are usually treated by embolization.
A
  1. (A) Carotid body tumors are usually 3–4 mm in
    size and are located at the carotid bifurcation.
    They arise from nests of chemoreceptor cells
    of neuroectodermal origin (carotid body). In
    normal individuals, the carotid body responds
    to a fall in PO2 and pH and to a rise in PCO2 and
    temperature to cause an increase in cardiac contraction, heart rate, and respiratory rate. Carotid
    body tumors are uncommon, slow growing, and
    highly vascular. Although large tumors may
    cause compression of the vagus or hypoglossal
    nerves, most tumors present as a palpable painless mass at the carotid bifurcation. The treatment is definitely excision whenever possible.
139
Q
  1. A middle-aged man complains of shortdistance claudication in the right thigh. The
    angiogram shows a right common iliac artery
    stenosis of 90% over a short segment. What is
    the treatment of choice?
    (A) Aortofemoral bypass
    (B) Left-to-right fermorofemoral bypass
    (C) Iliofemoral bypass
    (D) PTA and stent placement
    (E) Axillofemoral bypass
A
  1. (D) PTA is technically successful in approximately 90% of iliac lesions with good patency
    rates. It is more successful for single short
    stenoses rather than multiple long stenosis or
    occlusions. The advantages of PTA is that it is
    less invasive than surgery, has a lower initial
    cost, has a shorter hospital stay, and lower morbidity, enables an earlier return to full activity,
    and the procedure can be repeated without an
    increase in morbidity or a decrease in clinical
    result. It is particularly useful for patients who
    are at high operative risks. The ideal procedure
    would be and angioplasty and stent placement.
140
Q
52. A 65-year-old man with hypertension and a
blood pressure of 190/105 mm Hg has unilateral renal artery stenosis. What is the best diagnostic test to determine the physiologic
significance of the lesion?
(A) Aortography
(B) Renal scan
(C) Renal ultrasound
(D) Renal vein renin assay
(E) Rapid-sequence intravenous pyelogram
A
  1. (D) Aortography and renal ultrasound can detect
    the presence of renal artery stenosis, but they do
    not determine the functional significance of the
    lesion. IVP is not a sensitive enough test to detect
    the presence of renal artery stenosis. Arenal scan
    can show decreased flow (uptake) or decreased
    function of the affected kidney, but it, too, lacks
    sensitivity. The assessment of renal vein renin
    levels is a good diagnostic test to determine the
    physiologic significance of renal artery stenosis.
    It indicates whether the stenosis is significant
    enough to decrease the glomerular filtration rate
    and cause the release of renin. In addition, the
    opposite kidney should have suppression of
    renin secretion.
141
Q
  1. A young college student injures his left knee
    while playing football and is unable to bear
    weight. The provisional x-ray report indicates
    that there are no fractures seen. He is discharged home but presents the next morning to
    the emergency department with a severely
    swollen, painful left knee and severe pain in the
    foot. On examination, the foot is pale, cold, and
    pulseless. What is the most likely diagnosis?
    (A) Traumatic deep vein thrombosis
    (B) Gastrocnemius muscle tear
    (C) Traumatic arteriovenous fistula
    (D) Posterior knee dislocation with
    thrombosed popliteal artery
    (E) Traumatic sciatic neuropathy
A
  1. (D) Normal radiographic findings in the presence of severe knee trauma should raise suspicion for posterior dislocation of the knee, which
    is often associated with popliteal artery thrombosis. A careful vascular examination should,
    therefore, be made in such a situation. The presence of pain, pallor, and pulselessness (three of
    the five p’s) is indicative of severe ischemia. This
    patient should undergo urgent exploration for
    vascular repair. The other options are unlikely to
    cause the signs and symptoms presented.
142
Q
  1. An elderly patient complains of recurrent
    episodes of amaurosis fugax. This is attributable
    to microembolization of which of the following?
    (A) Facial artery
    (B) Retinal artery
    (C) Occipital artery
    (D) Posterior auricular artery
    (E) Superficial temporal artery
A
  1. (B) Amaurosis fugax, one type of TIA, is a manifestation of carotid bifurcation atherosclerotic
    disease. It is manifested by unilateral blindness,
    being described by the patient as a window
    234 10: Vascular
    shade across the eye, lasting for minutes or
    hours. It is caused by microemboli from a carotid
    lesion lodging in the retinal artery, the first intracerebral branch of the internal carotid artery.
143
Q
  1. A 65-year-old woman television technician
    undergoes femoral embolectomy and leg fasciotomy. Following surgery, she is noted to
    have oliguria, and her urine is red. What is the
    most probable diagnosis?
    (A) Hematuria secondary to heparin
    (B) Embolus of the renal artery
    (C) Myoglobinuria
    (D) Retroperitoneal hematoma
    (E) Hemoglobinuria
A
  1. (C) Patients with sudden severe ischemia are
    prone to “ischemia-reperfusion” syndrome. With
    revascularization, there is sudden release of the
    accumulated products of ischemia into the circulation; namely, potassium, lactic acid, myoglobin, and cellular enzymes. Hyperkalemia,
    metabolic acidosis, and myoglobinuria (red
    urine, clear plasma) are the key features of
    the syndrome. Renal tubular acidosis results
    in myoglobin deposition in the renal tubules.
    Anticipation and early recognition require the
    induction of diuresis with mannitol, alkalinization of the urine to avoid precipitation of myoglobin in the renal tubules, and correction of
    hyperkalemia.
144
Q
56. A 24-year-old woman on oral contraceptive
pills develops an episode of deep vein thrombosis that is adequately treated with anticoagulation. She is at increased risk of developing
which of the following?
(A) Recurrent foot infections
(B) Claudication
(C) Pulmonary embolism
(D) Postphlebetic syndrome
(E) Superficial varicose veins
A
  1. (D) Despite receiving optimal treatment for
    DVT, approximately 50% of the patients will
    develop the post-thrombotic syndrome. The
    recanalization of the deep veins will result in
    deformity and subsequently incompetence of
    the affected venous valves. Although patients
    with DVT can develop infections secondary to
    edema, these are usually located about the
    ankle and resolve with adequate treatment.
    Patients adequately treated for DVT are not at
    increased risk of developing pulmonary embolus. Neither the arterial circulation nor the
    superficial venous system are affected by the
    development of DVT. Young patients with
    iliofemoral thrombosis are best managed by
    thrombolytic infusion, which has been shown
    to preserve valvular function and decrease the
    incidence of postphlebitic syndrome.
145
Q
  1. A72-year-old businessman undergoes a femoralto-posterior tibial in situ bypass graft for a
    nonhealing foot ulcer. During routine follow-up
    examination 4 years later, the graft is found to be
    occluded. The cause of his graft failure is most
    probably secondary to which of the following?
    (A) Progression of atherosclerosis
    (B) Technical error
    (C) Retained valve in the conduit
    (D) Venous aneurysm
    (E) Intimal hyperplasia
A
  1. (A) The causes of graft failure can be divided
    into early and late. Although early failure of
    vein grafts is usually attributed to either technical error or inadequate outflow tract, late failure is usually related to progressive proximal
    or distal atherosclerotic disease. Other less
    common causes of late graft failures include—
    local stenotic areas from trauma or endothelial
    damage, valve stenosis from fibrosis, and venous
    aneurysms and subsequent thrombosis. Intimal
    hyperplasia is a rare cause of late failure.
146
Q
  1. A60-year-old woman has an asymptomatic right
    carotid bruit. A carotid duplex scan shows no
    evidence of significant carotid bifurcation disease but reveals reversal of flow in the right vertebral artery. What is the most likely diagnosis?
    (A) Stenosis of the origin of the common
    carotid artery
    (B) Stenosis of the vertebral artery
    (C) Stenosis of the subclavian artery
    (D) Stenosis of the external carotid artery
    (E) Stenosis of the intracranial portion of
    the internal carotid artery
A
  1. (C) Occlusion or stenosis of the subclavian
    artery proximal to the origin of the vertebral
    artery results in the “subclavian steal” syndrome. In response to decreased pressure in the
    distal subclavian artery, especially in instances
    in which increased perfusion is needed, there is
    reversal of flow in the vertebral artery. The clinical picture is that of vertebrobasilar symptoms
    in association with upper extremity exercise.
    Although this phenomenon is sometimes seen
    on duplex scanning or angiography, evolution
    into a clinical syndrome is relatively rare. The
    other mentioned options do not result in retrograde flow in the vertebral artery.
147
Q
  1. A newborn girl with family history of lymphedema is noted to have bilateral lower
    extremity swelling. What is the diagnosis?
    (A) Secondary lymphedema
    (B) Lymphedema praecox
    (C) Milroy disease
    (D) Lymphedema tarda
    (E) Meigs’s syndrome
A
  1. (C) Lymphedema is classified by etiology—primary versus secondary. Primary lymphedema
    is divided into congenital, praecox, and tarda,
    depending on the age of onset. The diagnosis of
    Milroy disease is reserved for patients with
    familial lymphedema in which clinical factors
    are present at birth or noticed soon thereafter.
    Lymphedema is classified as praecox if the age
    of onset is between 1 and 35 years. Meigs’
    disease is the familial form of primary lymphedema praecox. If the onset of primary lymphedema is after 35 years of age, it is called
    lymphedema tarda. Secondary lymphedema
    usually results from a disease process that
    causes obstruction of the lymphatic system.